Questions

अब Quizwiz के साथ अपने होमवर्क और परीक्षाओं को एस करें!

Which developmental theorist created the psychosocial theory of development? a) Piaget b) Freud c) Erikson d) Maslow

Erikson Correct Explanation: Erikson's psychosocial theory of development encompassed social and cultural influences.

Due to a patient's reluctance to begin hormone therapy, the advance nurse practitioner prescribes venlafaxine (Effexor) to decreases the risk of which of the following? a) Heart attack b) Blood clots c) Hot flashes d) Stroke

Hot flashes Explanation: Problematic hot flashes have been treated with low-dose venlafaxine (Effexor) as an alternative therapy for hot flashes. Hormone therapy decreases the risk of hot flashes, but it increases the risk of stroke, heart attack, and blood clots.

The nurse is assessing the development of an 18-month-old. The child should be able to: a) use a spoon with little spilling. b) say three words. c) throw a ball overhand. d) build a tower of four cubes.

say three words. Correct Explanation: By age 18 months, 90% of children can say three words. Typically, a child 23 months of age can build a tower of four cubes. The ability to use a spoon or fork with little spilling is accomplished by the age of 20 months. Throwing a ball overhand typically is achieved by age 3.

A mother expresses concern because her 3-year-old son frequently fondles his penis. The mother does not know the best approach for the child's behavior. What is the nurse's best response to the mother? a) "This behavior is normal for a child of his age." b) "You should discourage this behavior now before it worsens as he gets older." c) "We should obtain a urine sample to assess for an infection." d) "This is a strong sign that he is ready for toilet training."

"This behavior is normal for a child of his age." Correct Explanation: Children ages 1 to 3 years enjoy fondling their genitals. Punishment for genital fondling may lead to guilt and shame regarding sexual behavior later in life.

You are a student nurse visiting a day care. Your focus today is to observe "object permanence". According to Piaget, "object permanence" does not occur until which of the following ages? a) 24 months b) 12 months c) 6 months d) 8 months

12 months Explanation: According to Piaget, at 12 to 18 months the child is able to recognize the permanence of objects, that the object continues to exist even if it is out of sight.

A new mother asks, "When will the soft spot near the front of my baby's head close?" The nurse should tell the mother the soft spot will close in about: a) 2 to 3 months. b) 12 to 18 months. c) 9 to 10 months. d) 6 to 8 months.

12 to 18 months. Correct Explanation: Normally, the anterior fontanel closes between ages 12 and 18 months. Premature closure (craniostenosis or premature synostosis) prevents proper growth and expansion of the brain, resulting in an intellectual disability. The posterior fontanel typically closes by ages 2 to 3 months.

A client tells the nurse that she has had sexual contact with someone whom she suspects has genital herpes. The nurse should instruct the client to: a) continue sexual activity unless lesions are present. b) report any difficulty urinating. c) anticipate lesions within 25 to 30 days. d) drink extra fluids to prevent lesions from forming.

report any difficulty urinating. Explanation: The client should be encouraged to report painful urination or urinary retention. Lesions may appear 2 to 12 days after exposure. The client is capable of transmitting the infection even when asymptomatic, so a barrier contraceptive should be used. Drinking extra fluids will not stop the lesions from forming

College freshman are participating in a study abroad program. When teaching them about hepatitis B, the nurse should instruct the students on the need for: a) safe sexual practices. b) water sanitation. c) single dormitory rooms. d) vaccination for hepatitis D.

safe sexual practices. Correct Explanation: Hepatitis B is considered a sexually transmitted disease, and students should observe safe-sex practices. Poor sanitary conditions in underdeveloped countries relate to spread of hepatitis A and E. Focusing on routes of transmission and avoidance of infection can prevent the spread of hepatitis; isolation in single rooms is not required. There is no vaccine for hepatitis D

You are the pediatric nurse caring for Beth, a 13-month-old who is admitted with a respiratory infection. Her mother says she wants her child to develop in the best way possible. She asks you, "What kind of self-concept should Beth have at her age?" Your best answer is which of the following? a) "Her sense of self is very consolidated at this age." b) "She has developed no self-concept at this age." c) "The differences between self and others are strong at this age." d) "She has a beginning differentiation of self and non-self."

"She has a beginning differentiation of self and non-self." Explanation: The newborn has no self-concept at birth. In late infancy, a baby starts to differentiate between self and non-self. In childhood, differences between self and others are strong. During adolescence one's sense of self becomes very consolidated.

A 6-month-old child is taken to the pediatrician, and the mother states that the child is not growing like other children of similar age in families she knows. The birth weight of the child was 7 pounds 11 ounces, (3,495 g) and the current weight is 11 pounds 2 ounces (5,057 g). Based on these findings, the nurse tells the mother: a) "Your infant's weight is within the normal range based on the infant's age. No further action is required." b) "You should not compare your infant's weight based on other infants of the same age because each child's weight gain differs. No further action is required." c) "Your infant's weight is below the normal range based on the infant's age. Let's start with a few questions regarding your infant's eating habits". d) "Your infant's weight is above the normal range based on the infant's age. Let's start with a few questions regarding your infant's eating habits".

"Your infant's weight is below the normal range based on the infant's age. Let's start with a few questions regarding your infant's eating habits". Explanation: Birth weight usually doubles by age 6 months and triples by age 1 year. Therefore, this infant should weigh 14 lb (6.4 kg). Watchful waiting or no action is detrimental to the infant's growth and development. Comparison to other children is not helpful. Asking about the child's eating habits will help the nurse get a better understanding of potential causes of the low birth weight. The parents should be advised that the birth weight is below normal.

A nurse is assessing the respiratory rate of a sleeping infant. Which of the following would the nurse document as a normal finding? a) 12 to 20 breaths per minute b) 60 to 80 breaths per minute c) 80 to 100 breaths per minute d) 30 to 60 breaths per minute

30 to 60 breaths per minute Explanation: When assessing the respiratory rate of an infant, the nurse knows that the normal respiratory rate of an infant at rest is approximately 30 to 60 breaths per minute. The normal respiratory rate of an adult is 12 to 20 breaths per minute. A respiratory rate of 60 to 80 breaths per minute or 80 to 100 breaths per minute is abnormal and is not seen in infants or adults when they are at rest. Tachypnea is an abnormally fast respiratory rate, usually above 20 breaths per minute in the adult, whereas bradypnea is an abnormally slow respiratory rate, usually less than 12 breaths per minute in the adult.

Which example best supports the diagnosis of Sexual Dysfunction: Dyspareunia? a) A 50-year-old woman with a history of stroke is afraid to have sex with her partner for fear it will elevate her blood pressure. b) A 50-year-old woman in the process of menopause has pain and burning during intercourse. c) A 39-year-old alcoholic woman is no longer interested in having sex with her partner. d) A patient with a colostomy believes she cannot have a sexual relationship with her husband because he will be repulsed by her stoma.

A 50-year-old woman in the process of menopause has pain and burning during intercourse. Correct Explanation: Dyspareunia refers to pain and burning during intercourse. This is a common cause of sexual dysfunction, especially during menopause. A colostomy, fear of blood pressure elevation, and lack of interest in sex may lead to the nursing diagnosis of Sexual Dysfunction, but not related to dyspareunia.

You are reviewing developmental theories for class discussion. You know that Freud's theory emphasizes the effect of instinctual human drives on behavior. He identified the underlying stimulus for human behavior as sexuality, which he called libido. Freud defined libido as which of the following? a) Seeking sexual gratification b) Pure genital gratification c) General pleasure-seeking instincts d) Seeking sexual stimulation

General pleasure-seeking instincts Explanation: Freud defines libido as general pleasure-seeking instincts rather than purely genital gratification.

Which of the following statements explains how pregnancy affects the growth and development of the adolescent? a) Affects the normal physical and maturation processes b) Interferes with ability to care for newborn due to loss of innocence c) Inhibits further education due to caring for infant d) Causes damage to self-identity requiring psychological counseling

Affects the normal physical and maturation processes Correct Explanation: Pregnancy affects the normal physical and maturation processes of the adolescent. Psychological counseling is not indicated unless the adolescent shows a need for counseling. Loss of innocence does not interfere with the ability to care for an infant. The adolescent can continue education while caring for an infant.

When do most perinatal HIV infections occur? a) After exposure during delivery b) Through casual contact c) Through breastfeeding d) In utero

After exposure during delivery Explanation: Mother-to-child transmission of HIV-1 may occur in utero or through breastfeeding, but most perinatal infections are thought to occur after exposure during delivery.

There is a predominant pattern of variations that occur during the male sexual response. Which of the following occurs during the orgasm phase? a) Thickening of the the scrotal skin. b) Expulsive contractions of the urethra. c) Rapid loss of vasocongestion. d) Thickening of the penis at the coronal ridge.

Expulsive contractions of the urethra. Correct Explanation: Expulsive contractions of the entire length of the urethra occur during orgasm. Rapid loss of vasocongestion is seen in the resolution phase, which is the fourth and final phase immediately after orgasm. Thickening of the scrotal skin occurs during the excitement phase. During the plateau phase, the penis circumference at the coronal ridge thickens.

A nurse caring for the skin of patients of different age groups should consider which accurately described condition? a) An adolescent's skin ordinarily has enlarged sebaceous glands and increased glandular secretions. b) An infant's skin and mucous membranes are protected from infection by a natural immunity. c) Secretions from skin glands are at their maximum from age 3 on. d) The skin becomes thicker and more leathery with aging and is prone to wrinkles and dryness.

An adolescent's skin ordinarily has enlarged sebaceous glands and increased glandular secretions. Correct Explanation: Adolescents have enlarges sebaceous glands and increased glandular secretions, which predisposes them to acne. Infants have natural immunities, but not pertaining to the mucous membranes. Secretions from skin glands occur later than age 3 months. While the skin may have more wrinkles as a person ages, the skin actually becomes thinner with age

The nurse recommends a barrier method of contraception for a patient who is concerned about the side effects of hormonal contraception. What method might the nurse suggest? a) Abstinence b) Sterilization c) Cervical cap d) Norplant system

Cervical cap Correct Explanation: Barrier methods of birth control help in preventing pregnancy by using a device to block sperm from fertilizing an eggs. Barrier methods include the cervical cap, condom, diaphragm, and vaginal sponge used in combination with a spermicidal agent. Abstinence means having no sexual intercourse. Norplant is a hormonal method of birth control. Sterilization is a surgical method of birth control

A woman infected with HIV comes into the clinic. What symptoms may be the focus of a medical complaint in women infected with HIV? a) Rashes on the face, trunk, palms, and soles b) Muscle and joint pain c) Gynecologic problems d) Weight loss

Gynecologic problems Correct Explanation: In women with HIV, gynecologic problems, such as abnormal results of Papanicolaou tests, genital warts, pelvic inflammatory disease, and persistent vaginitis may be the focus of a majority of complaints. Acute retroviral syndrome (viremia) may be the chief complaint in one third to more than one half of those infected, not necessarily women. Its manifestations include rashes, muscle and joint pain, and weight loss

Which interventions are appropriate when developing a plan of care for promoting the development of a preschooler? Select all that apply. a) Help the parents understand their child's behavior. b) Determine the child's future development. c) Identify deviations from normal growth and development patterns. d) Provide anticipatory guidance for parents. e) Send the child to a day care center.

Help the parents understand their child's behavior. • Identify deviations from normal growth and development patterns. • Provide anticipatory guidance for parents. Correct Explanation: Goals for promoting healthy development in preschoolers include anticipatory guidance, helping parents understand their child's behavior, identifying deviations from the norm, and assessing parent-child interaction. No one can assess or determine the child's future development, and trying to do so can limit the potential the child may achieve. Although learning to interact with others is important, sending the child to a day care center is not essential to promote healthy development. The nurse can encourage the parents to provide opportunities for the child to play with others

A patient reports a pain rating of 6 (on a 1 to 10 pain scale, with 10 being the worst possible pain) when the nurse examiner gently moves the cervix. This finding is most consistent with which of the following conditions? a) Uterine prolapse b) Rectocele c) Endometriosis d) Pelvic infection

Pelvic infection Explanation: Pain on gentle movement of the cervix is called a positive chandelier sign or positive cervical motion tenderness and usually indicates a pelvic infection.

Parents of a 2-year-old child with chronic otitis media are concerned that the disorder has affected their child's hearing. Which behavior suggests that the child has a hearing impairment? a) Using gestures to express desires b) Stuttering c) Babbling continuously d) Playing alongside rather than interacting with peers

Using gestures to express desires Explanation: Using gestures instead of verbal communication to express desires — especially in a child older than age 15 months — may indicate a hearing or communication impairment. Stuttering is normal in children ages 2 to 4, especially boys. Continuous babbling is a normal phase of speech development in young children. In fact, its absence, not presence, would be cause for concern. Parallel play — playing alongside peers without interacting — is typical of toddlers. However, in an older child, difficulty interacting with peers or avoiding social situations may indicate a hearing deficit

A nurse is assessing a child who has a mild intellectual disability. The best indication of how this child is progressing can be obtained by observing him: a) in the clinic with his mother. b) at school with his teacher. c) playing soccer with his friends. d) at home with his family.

at school with his teacher. Explanation: Watching the child relate to his teacher and schoolwork is the best indication of how he is progressing. School involves interacting with a person who is not a relative and in a situation that is not totally familiar. Observing the client in situations with family and friends shows social relationships but does not indicate how the child is learning new intellectual skills.

A 30-year-old client is being treated for epididymitis. Teaching for this client should include the fact that epididymitis is commonly a result of a: a) protozoon. b) virus. c) parasite. d) sexually transmitted infection

sexually transmitted infection. Explanation: Among men younger than age 35, epididymitis is most frequently caused by a sexually transmitted infection. Causative organisms are usually chlamydia or Neisseria gonorrhoeae. The other major form of epididymitis is bacterial, caused by the Escherichia coli or Pseudomonas organisms. The nurse should always include safe sex teaching for a client with epididymitis. The client should also be advised against anogenital intercourse because this is a mode of transmission of gram-negative rods to the epididymis

Donald, a newly graduated nurse, is working in a pediatric clinic. Today he sees Karen, a 1-month-old infant, and her single mother. The mother is estranged from Karen's father but is marrying her high school sweetheart next week. Donald remembers from nursing school that which of the following will affect an infant's growth and development? Choose all that apply. a) Genetics of mother b) Mother's prenatal nutrition c) Environment of infant d) Father's gender identity e) Mother's gender identity

• Genetics of mother • Environment of infant • Mother's prenatal nutrition Correct Explanation: Growth and development might be facilitated or delayed by genetic heredity; prenatal, individual, and caregiver factors; and environment and nutrition. Gender identity is the inner sense a person has of being male or female and should not effect growth and development of an infant.

The nurse working at a family planning clinic evaluates a client's risk for complications from oral contraceptives. Which of the following places the client at highest risk? a) History of diabetes b) Age less than 35 c) Drinks 1 glass of wine daily d) Smokes 1 pack cigerettes per week

Smokes 1 pack cigerettes per week Correct Explanation: Complications for oral contraceptives include an increased incidence of blood clots, heart attacks, and stroke, especially in women who smoke.

Which statement by a parent whose child was just diagnosed with pediculosis capitis (head lice) demonstrates an understanding of the safety and efficacy of the common medications used to treat the infection? a) "I am going to request a prescription for lindane since it works the best." b) "I like the OTC lice treatments because you can give a second treatment the next day if any lice remain." c) "After I shampoo, I will use the special comb to get the nits out." d) "Most OTC lice treatments are100% effective at killing all the eggs."

"After I shampoo, I will use the special comb to get the nits out." Correct Explanation: The makers of many of the pediculicides recommend manual removal of the nits following treatment with an extra-fine-tooth comb. None of the pediculicides are 100% effective in killing all the eggs. The FDA has issued a warning regarding the use of lindane because of the potential for neurotoxicity. Clients are treated with lindane only when the benefits outweigh the risks. Lice treatments may be repeated in 7 to 10 days; the next day is too soon.

A woman with cystitis is to take a 10 day prescription of an antibiotic. The client asks the nurse if she can continue to have sexual intercourse. The nurse should tell the client?: a) "As long as you are comfortable, you can have intercourse as often as you wish, but be sure to urinate within 15 minutes after intercourse." b) "Avoid intercourse until you have completed the antibiotic therapy, and then limit intercourse to once a week." c) "Limit intercourse to once a day in the early morning after your bladder has rested." d) "You can have intercourse as often as you wish, but be sure your partner uses a condom."

"As long as you are comfortable, you can have intercourse as often as you wish, but be sure to urinate within 15 minutes after intercourse." Explanation: Intercourse is not contraindicated in cystitis. Voiding immediately after intercourse flushes bacteria from the urethra, which should help prevent recurrence.

A first-time mother is concerned that her 6-month-old infant is not gaining enough weight. What should the nurse tell the mother? a) "Birth weight doubles by 3 months of age." b) "The baby will eat what he needs." c) "Birth weight doubles by 6 months of age." d) "You need to make sure the baby finishes each bottle."

"Birth weight doubles by 6 months of age." Correct Explanation: A general growth parameter is that the birth weight doubles in 6 months and triples in a year. Telling the mother that the baby will eat what he needs is not appropriate. The nurse needs to investigate whether the baby's weight is within the normal parameters of infant weight gain. A bottle-fed baby should not be forced to complete the bottle because this may contribute to obesity.

A mother is concerned that her neonate, who was born without complications at 38 weeks, isn't eating enough and will lose too much weight. The mother states, "He only breast-feeds for about 3 minutes on one side." Which instruction should the nurse provide to this mother? a) "I am concerned, too, and will notify the pediatrician immediately." b) "Don't worry. When he's hungry, he'll eat. You'll see; it will be fine." c) "I understand your concern, but he has stored nutrients before birth just for this reason." d) "It's important that he doesn't lose too much weight. We should start him on formula after each feeding."

"I understand your concern, but he has stored nutrients before birth just for this reason." Correct Explanation: Telling the client that her neonate has stored nutrients is the most appropriate response. Neonates who are born at term without complications have stores of brown fat located on the vital organs. These stores will provide the neonate with the needed calories until lactation is well established. Telling the client not to worry and saying things will be fine ignores the mother's concerns. Starting the neonate on formula and notifying the physician are inappropriate at this time

The mother of a 17-year-old girl with Down syndrome tells the nurse that her daughter recently stated that she has a boyfriend. The mother is concerned that her daughter might become pregnant. Which is the most appropriate suggestion by the nurse? a) "This may be difficult, but you may want to suggest that your daughter break off the relationship." b) "Women with Down syndrome are infertile, so you do not need to worry about her getting pregnant." c) "I understand your concern; you may want to start your daughter on long-acting contraception." d) "I understand your concern; you may want to enroll your daughter in an abstinence program."

"I understand your concern; you may want to start your daughter on long-acting contraception." Correct Explanation: Children with Down syndrome range from severely intellectual disability to low average intelligence, Thus the adolescent's ability to make informed choices regarding sexual activity is limited. Long-acting contraception, such as an intrauterine device or a progestin implant, greatly reduces the risk of unwanted pregnancy. Most women with Down syndrome are fertile; however, children born to women with Down syndrome often have congenital defects. An abstinence program may not be effective due to the intellectual level of children with Down syndrome. Suggesting that the adolescent break off the relationship does not ensure that she will.

A school nurse is conducting a safety seminar with 6th-grade students. Which of the following teaching points is of primary importance? a) "A wood-burning fireplace is a major fire risk, and it shouldn't be used unless necessary." b) "If your clothes should catch on fire, remember to stop, drop, and roll." c) "Remember that cigarettes don't just hurt your lungs, they can be a cause of fires." d) "Make sure that you have smoke detectors in your house and that they're in working order."

"Make sure that you have smoke detectors in your house and that they're in working order." Explanation: There are fire risks associated with cigarettes, clothing combustion, and fireplaces that should be addressed, but the paramount fire-safety issue is smoke detectors, since approximately half of home fire deaths occur in a home without a smoke detector.

A parent asks the nurse about the nutritional needs of her toddler. Which response by the nurse would be most appropriate? a) "Toddlers usually do not have a good appetite." b) "Toddlers have definite food preferences." c) "Toddlers are inquisitive, willing to try new foods." d) "Toddlers usually consume large quantities of milk."

"Toddlers have definite food preferences." Explanation: Toddlers have definite food preferences, typically wanting the same food item for several days in a row. Because toddlers experience a slow and steady growth rate, they usually have a good appetite. Toddlers should consume 2 to 3 servings of milk per day. The majority of their nutrients should come from table foods. Toddlers typically are not interested in trying new foods.

The nurse is providing health screening for adolescent girls. Which of the following adolescent girls does the nurse identify as highest risk for an unplanned pregnancy? Select all that apply. a) An adolescent girl with low educational achievement b) An adolescent girl dating an older boy c) An adolescent girl who believes in abstinence d) An adolescent girl with low self-esteem e) An adolescent girl living in poverty

An adolescent girl with low self-esteem • An adolescent girl living in poverty • An adolescent girl dating an older boy • An adolescent girl with low educational achievement Correct Explanation: Young women may try to use a pregnancy to escape a poor living situation. Those with low education and literacy levels may not possess the knowledge or information needed to protect themselves from unwanted pregnancies. A young woman with low self-esteem may be pressured into a sexual relationship, especially when involved with an older boy, resulting in an unwanted pregnancy.

Katrina Sterrett, a 26-year-old preschool teacher, is being seen by a physician who is part of the internist group where you practice nursing. She is undergoing her annual physical and is having many lab tests done as a condition of her employment and upcoming wedding. She is returning for her results and is devastated to learn that she has the sexually transmitted infection, gonorrhea. What would contribute to her ignorance of her condition? a) Being sexually inactive b) Knowing the signs and symptoms of STIs c) All options are correct. d) Being asymptomatic

Being asymptomatic Correct Explanation: Many women who have gonorrhea are asymptomatic, a factor that contributes to the spread of the disease. Knowing the signs and symptoms of STIs will not help with an asymptomatic disease. Being sexually inactive currently will not prevent having been infected with a disease in the past. All options are not correct

A 20-year-old nulligravid client expresses a desire to learn more about the symptothermal method of family planning. Which information would the nurse include in the teaching plan? a) Cervical mucus is carefully monitored for changes. b) The male partner uses condoms for significant effectiveness. c) This method has a 50% failure rate during the first year of use. d) Couples must abstain from coitus for 5 days after the menses.

Cervical mucus is carefully monitored for changes. Explanation: The symptothermal method is a natural method of fertility management that depends on knowing when ovulation has occurred. Because regular menstrual cycles can vary by 1 to 2 days in either direction, the symptothermal method requires daily basal body temperature assessments plus close monitoring of cervical mucus changes. The method relies on abstinence during the period of ovulation, which occurs approximately 14 days before the beginning of the next cycle. Abstinence from coitus for 5 days after menses is unnecessary because it is unlikely that ovulation will occur during this time period (days 1 through 10). Typically, the failure rate for this method is between 10% and 20%. Although a condom may increase the effectiveness of this method, most clients who choose natural methods are not interested in chemical or barrier types of family planning.

A nurse is counseling a couple who expressed that they are having difficulty conceiving. Which of the following could the nurse consider as a cause? a) Masturbating b) Changing developmental stage c) Having sex during menstruation d) "Petting" prior to intercourse

Changing developmental stage Explanation: Changing developmental stages, such as menopause, can interfere with a couple's normal sexual expression. Having sex during menstruation, masturbation, and "petting" or simple stroking as part of foreplay do not affect a couple being able to conceive.

A nurse is working with a child on a pediatric unit. Based on the age of the child, the nurse understands the child should be in Piaget's Concrete Operational stage. Which of the following activities should the nurse plan to include in the plan of care to evaluate if the child has achieved this stage? a) Playing monopoly with other children b) Using pictures for language development c) Classifying and sorting coins d) Using utensils during meals

Classifying and sorting coins Correct Explanation: Piaget's Concrete Operational stage is when children learn by manipulating objects and can classify them by two or more characteristics. In this example, coins can be classified by color and size. Using utensils at meals is an example of Piaget's Sensorimotor stage. Using pictures for language development is an example of Piaget's Preoperational stage. Playing Monopoly with other children is an example of Piaget's Formal Operational stage and is the most advanced stage in this theory.

A nurse suspects that a client may be experiencing post-traumatic stress disorder (PTSD). Which assessment finding would help support the nurse's suspicion? a) Client had a foot amputated due to diabetes. b) Client was a victim of a rape and beating. c) Client was just told that he has lung cancer. d) Client recently lost a spouse to a chronic illness.

Client was a victim of a rape and beating. Correct Explanation: PTSD results from an overwhelming event that is outside the range of normal human experience. Rape and beating would be one such example. The diagnosis of lung cancer would most likely cause anxiety and possibly depression. Amputation of a body part or loss of a spouse would most likely lead to depression.

A 16-year-old has been diagnosed with polycystic ovarian syndrome (PCOS). Which statement indicates the need for more teaching? a) "Untreated PCOS will make getting pregnant impossible." b) "I am at risk for type 2 diabetes." c) "High levels of male hormones contribute to my PCOS." d) "Maintaining a healthy weight is an important part of my treatment plan."

Correct response: "Untreated PCOS will make getting pregnant impossible." Explanation: While pregnancy may be difficult for some clients, the nurse must work to prevent the false conception that a sexually active teen with PCOS does not need to use a reliable form of birth control. PCOS is associated with high levels of androgens and excessive insulin. It is the excess insulin that is thought to increase androgen production. Clients with PCOS are at risk for type 2 diabetes. Initial treatment focuses on weight management and exercise. These measures often reduce insulin production and restore menstrual cycles.

Which of the following is a failure of one or both or both of the testes to descend into the scrotum? a) Cryptorchidism b) Hydrocele c) Phimosis d) Varicocele

Cryptorchidism Correct Explanation: Cryptorchidism is the failure of one or both of the testes to descend into the scrotum. Hydrocele is a collection of fluid, generally in the tunica vaginalis of the testes. Varicocele is an abnormal dilation of the veins of the pampiniform venous plexus in the scrotum. Phimosis is a condition in which the foreskin is constricted so that is cannot be retracted over the glans.

After talking with the parents of a child with Down syndrome, the nurse should help the parents establish which goal? a) Encourage self-care skills in the child. b) Encourage more lenient behavior limits for the child. c) Teach the child something new each day. d) Achieve age-appropriate social skills.

Encourage self-care skills in the child. Correct Explanation: The goal in working with children with intellectual disabilities is to train them to be as independent as possible, focusing on developmental skills. The child may not be capable of learning something new every day but needs to repeat what has been taught previously. Rather than encouraging more lenient behavior limits, the parents need to be strict and consistent when setting limits for the child. Most children with Down syndrome are unable to achieve age-appropriate social skills due to their disability. Rather, they are taught socially appropriate behaviors.

To meet the developmental needs of an 8-year-old child who is confined to home with osteomyelitis, what goal should the nurse include in the care plan? a) Encourage the parents to stay with the child. b) Talk to the child about his interests twice daily. c) Allow siblings to visit freely throughout the day. d) Encourage the child to communicate with schoolmates.

Encourage the child to communicate with schoolmates. Correct Explanation: Encouraging contact with schoolmates allows the school-age child to maintain and develop socialization with peers, an important developmental task of this age group. Although having family visits and interacting with the child are important, they do not meet the child's developmental needs. Talking to the child about his interests is important, but encouraging contact with schoolmates is crucial to maintain and develop socialization with peers.

Which diagnostic is indicated for postmenopausal bleeding? a) Endometrial biopsy b) CT c) MRI d) Ultrasound

Endometrial biopsy Explanation: For postmenopausal bleeding, an endometrial biopsy or a D & C is indicated.

The nurse is administering an 18-month-old boy's scheduled immunizations. The nurse would recognize that this child is most likely engaged in which of the following developmental tasks? a) Expressing his autonomy b) Developing trust c) Taking initiative d) Bonding with his father

Expressing his autonomy Explanation: The toddler (ages 1 to 3) enters Erikson's stage of autonomy versus shame and doubt, a developmental task that manifests as developing early independence in activities such as feeding and walking as well as expressing his wishes verbally. Bonding with his father and taking initiative are likely manifestations of this larger developmental task. The development of trust is primary during the first year of a child's life

You are a nurse working in a pediatric clinic. Today you are seeing Calvin Olson, who brought in his 6-month-old infant for a well-baby checkup. He asks what the common health problems are for this age. The nurse's correct reply includes which of the following? a) Communical diseases b) Respiratory infections c) Dental problems d) Food allergies

Food allergies Explanation: Gastroenteritis and food allergies are common health concerns during infancy. Dental problems, communical diseases, and respiratory problems are common health problems seen more often in toddlers and preschoolers.

Which of the following accurately describes a recommended guideline when administering oral medications to patients? a) Assume that the patient is the authority on whether or not the medication was swallowed. b) If a pill is dropped, it should be briefly immersed in saline to remove any dirt or germs. c) If a child refuses to take medication, the medication can be crushed and added to a small amount of food. d) If a patient vomits immediately after receiving oral medications, readminister the medication.

If a child refuses to take medication, the medication can be crushed and added to a small amount of food. Correct Explanation: Medication can be added to small amounts of food, but should not be added to liquids. If it is questionable whether the medication was swallowed, check the patient's mouth and cheeks. If a pill is dropped, it should be discarded, and if a patient vomits, notify the physician to see if the medication should be readministered.

According to Erikson's theory of development, chronic illness can interfere with which stage of development in an 11-year-old child? a) Intimacy versus isolation b) Trust versus mistrust c) Identity versus role confusion d) Industry versus inferiority

Industry versus inferiority Explanation: According to Erikson, an 11-year-old child is working through the stage of industry versus inferiority. Chronic illness may interfere with this stage of development in an 11-year-old child because the child may not be able to accomplish tasks, which prevents him from achieving a sense of industry. Intimacy is the developmental task of a young adult. Trust is the developmental task to be achieved during infancy. Identity is the developmental task of adolescence

Which of the following is inconsistent with acute otitis media? a) Conductive hearing loss may occur. b) A purulent exudate is usually present in the middle ear. c) It is a relatively uncommon childhood infection. d) The infection usually lasts less than 6 weeks.

It is a relatively uncommon childhood infection. Correct Explanation: Approximately 3 out of 4 children experience an ear infection by the time they are 3 years of age. The infection usually lasts less than 6 weeks. Conductive hearing loss may occur. A purulent exudate is usually present in the middle ear.

The nurse is advising an adolescent male about sexual myths that have him concerned. Which statements describe accurate patient teaching regarding these concerns? (Select all that apply.) a) A larger penis allows for a more satisfying sexual experience. b) Nocturnal emissions are normal in men of all ages. c) Nocturnal emissions are signs of a sexually transmitted disease. d) No male or female should feel pressured into sexual activity at any age. e) Nocturnal emissions indicate the existence of a sexual disorder. f) Masturbation or self-stimulation is a natural and healthy outlet for sexual urges.

Masturbation or self-stimulation is a natural and healthy outlet for sexual urges. • No male or female should feel pressured into sexual activity at any age. • Nocturnal emissions are normal in men of all ages. Explanation: Masturbation or self-stimulation is natural, no one should ever feel pressured into having sex, and nocturnal emissions are normal and occur in about 85% of men. Penis size has no effect on the satisfaction of the sexual experience.

Which of the following is the main cause of anemia in a patient with active uterine leiomyoma? a) Poor intake of foods containing iron b) Menorrhagia c) Hemolysis d) Pressure of the fibroid on the pelvic veins

Menorrhagia Correct Explanation: Uterine leiomyomas or fibroids cause menorrhagia, which in turn causes anemia. A poor dietary intake of iron does not cause anemia but aggravates the problem. Although there can be a feeling of pressure in the pelvic region, this does not cause anemia.

When assessing the chest of a 4-month-old infant, the nurse identifies the ratio of the anteroposterior-to-lateral diameter as 1:1. What action should the nurse take next? a) Inform the physician of the finding and obtain an order for a chest X-ray. b) Instruct the parents to bring the infant back in 1 month for reevaluation. c) No action is needed; this is a normal finding. d) Check the infant for signs of respiratory distress.

No action is needed; this is a normal finding. Correct Explanation: No action is needed by the nurse because in an infant, the anteroposterior diameter is normally equal that of the lateral diameter (a ratio of 1:1). As the infant reaches toddlerhood, the anteroposterior diameter becomes less than the lateral diameter.

Which suggestion would be most appropriate in helping parents to prepare their children for entry into school? a) Discuss school with the child if he asks about it. b) Orient the child to the school's physical environment. c) Offer to stay with the child for the first few days of school. d) Have an older sibling tell the child about school.

Orient the child to the school's physical environment. Correct Explanation: To help prepare a child to enter school, it is generally recommended that the child be taken to school to become oriented to the physical surroundings. This helps to lessen the child's concerns about what the school looks like and where he is going, thereby helping to alleviate some fears of the unknown. Older siblings are likely to criticize the younger child, possibly adding to the child's fears and feelings of insecurity. Staying with the child for a few days is not appropriate because it would foster dependence. The child needs to develop independence. The child may ask questions out of fear. Therefore, good preparation probably cannot be accomplished through discussions.

A 6-year-old child is admitted for an appendectomy. What is the most appropriate way for the nurse to prepare the child for surgery? a) Show the child a visual analog scale (VAS) based on a scale from 0 to 10. b) Explain how to use a patient-controlled analgesia (PCA) pump for pain control. c) Show the child a video about the surgery. d) Permit the child to play with the blood pressure cuff, electrocardiogram (ECG) pads, and a face mask.

Permit the child to play with the blood pressure cuff, electrocardiogram (ECG) pads, and a face mask. Correct Explanation: The best way to teach a child about surgery is through play. The nurse can let the child handle the items that will be used for monitoring, such as the blood pressure cuff and the ECG pads. The child will become more familiar with the face masks he sees the surgical team wearing in the operating room after playing with one and wearing it before surgery. A child of this age-group does not understand detailed explanations of how to use equipment, such as a PCA, a VAS, or even a video. The pain scale that should be used for children is the FACES scale.

Which of the following are age-related changes affecting the male reproductive system? a) Patency increases. b) Plasma testosterone levels decrease. c) Testes become soft. d) Prostate secretion increases.

Plasma testosterone levels decrease. Correct Explanation: Changes in gonadal function include a decline in plasma testosterone levels and reduced production of progesterone. The testes become smaller and more firm.

A male client informs the urology nurse that he is embarrassed because his wife rarely has time to reach sexual satisfaction during their encounters. He says he experiences orgasm as soon as he enters the wife's vagina. What is this condition best known as? a) Retarded ejaculation. b) Premature ejaculation. c) Impotence. d) Erectile failure.

Premature ejaculation. Correct Explanation: Premature ejaculation is when a man consistently achieves ejaculation or orgasm before or soon after entering the vagina. Erectile failure, also known as impotence, is the inability of a man to attain or maintain an erection to the extent that he cannot have satisfactory intercourse. Retarded ejaculation refers to a man's inability to ejaculate into the vagina or delayed intravaginal ejaculation

A nurse is planning care for a 10-year-old child in the acute phase of rheumatic fever. Which activity is most appropriate for the nurse to schedule in the care plan? a) Climbing on play equipment in the playroom b) Ambulating without restrictions c) Playing ping-pong d) Reading books

Reading books Correct Explanation: During the acute phase of rheumatic fever, the child should be placed on bed rest to reduce the workload of the heart and prevent heart failure. Therefore, an appropriate activity for this child would be reading books. Playing ping-pong, climbing on play equipment, and ambulating without restrictions are too strenuous during the acute phase.

During menstruation there is an absence of which hormone? a) FSH b) Estrogen c) LH d) Progesterone

Progesterone Explanation: During menstruation, there is an absence of progesterone. Estrogen is low, FSH is increased, and LH is low during menstruation.

To ensure that the breast-feeding neonate's weight loss remains within the expected parameter of 5% to 10%, the nurse should initially establish which type of feeding schedules? a) Maintain the neonate on an every-2-hours feeding schedule. b) Put the neonate to the mother's breast at least every 4 hours. c) Provide feeding on demand. d) Use supplementary bottles until the mother's milk comes in.

Provide feeding on demand. Explanation: Breast-feeding schedules should respond to the demands of the neonate, at a minimum of every 4 hours. A neonate may not be hungry or willing to eat every 2 hours. Every 4 hours may be too long for the neonate. Using supplementary bottles may interfere with the mother's milk production and cause nipple confusion

The nurse observes a hospitalized preschooler who clings excessively to his mother and uses infantile speech patterns. This child is exhibiting what type of behavior? a) Self-expression b) Separation anxiety c) Regression d) Negativism

Regression Explanation: Regression, behavior that is more characteristic of a younger age, can occur at any time in response to stressful circumstances. The most common regressive behaviors are excessive clinging to caregivers, loss of control over elimination, and the use of more infantile speech patterns. Negativism (characteristically expressed by saying no) and outbursts of temper result from the toddler's efforts at control over the environment. Separation anxiety in response to separation from the mother figure intensifies at about 18 to 24 months and can be as intense for the toddler as it was for the infant. Self-expression is a general term used to explain how an individual expresses his thoughts, feelings, and beliefs

A nurse is caring for a 9-year-old child who is shy and fearful. The nurse asks the child a question, but the child does not answer immediately. What is the best approach by the nurse to develop a therapeutic relationship with the child? a) Remain silent after asking a question. b) Use common clichés when asking questions. c) Explain the question with medical words. d) Tell the child the consequences of not answering.

Remain silent after asking a question. Correct Explanation: Silence offers an opportunity for the child to answer spontaneously and cautiously. More information is usually forthcoming if the nurse gives the child the opportunity to respond. All other choices are could potentially interfere with communication in the pediatric population.

A school nurse is teaching a class about sexually transmitted infections (STIs). Which statement is correct regarding STIs? a) The signs and symptoms of an STI are obvious. b) STIs disproportionately affect people with a lower socioeconomic status and education. c) STIs are most prevalent among teenagers and young adults. d) The incidence of STIs is decreasing due to limited sex partners.

STIs are most prevalent among teenagers and young adults. Correct Explanation: STIs are most prevalent among teenagers and young adults, and nearly two thirds of all STIs occur in people younger than 25 years. The incidence of STIs is increasing due to multiple sex partners and sexual activity at a younger age. STIs affect men and women of all backgrounds and economic levels.

As a grade school nurse, you speak to the sixth grade boys regarding their physical maturation and body changes during puberty. In your student education sessions, you discuss male anatomy and the appropriate titles and functions of each structure. Which structure is involved in keeping the testes at the necessary temperature to ensure sperm production? a) Epididymis b) Testicles c) Shaft d) Scrotum

Scrotum Correct Explanation: To maintain the temperature of the testes 3° cooler than body temperature, smooth and skeletal muscles in the scrotum pull the tissue toward the body when external temperatures are cold. On the other hand, the smooth muscles relax, causing the scrotum to become loose and hang away from the body, when environmental temperatures are hot. The scrotum is the divided sac of skin that contains the right and left testes, also called testicles. The location of the testes within the scrotal sac ensures optimum conditions for sperm production

Which of the following statements best describes the relationship between biologic sex and gender identity? a) Biologic sex is genetically determined but gender identity is chosen during adolescence. b) Biologic sex and gender identity are considered synonymous in nursing practice. c) Sex is chromosomally determined, while gender is a psychosocial construct. d) Biologic sex and gender identity are both modifiable by surgery and medical interventions.

Sex is chromosomally determined, while gender is a psychosocial construct. Explanation: Biologic sex is the term used to denote chromosomal sexual development. Gender identity is the inner sense a person has of being male or female. As such, biologic sex is amenable to medical interventions, but surgery and hormone therapy will not change an individual's inner sense of being male or female. Gender identity is not commonly thought to be chosen or objectively selected during adolescence.

The nurse talks to an adolescent about how she can tell her friends about her new diagnosis of diabetes. Which behavior by the adolescent indicates that the adolescent has responded positively to the discussion? a) She asks the nurse for material on diabetes for a school paper. b) She introduces the nurse to her friends as "the one who taught me all about my diabetes." c) She says, "I will try to tell my friends, but they will probably quit hanging out with me." d) She asks her friends what they think about someone who has a lifelong illness.

She introduces the nurse to her friends as "the one who taught me all about my diabetes." Correct Explanation: The ability to talk about her diabetes indicates that the adolescent feels good enough about herself to share her problem with her peers. Asking for reference material does not specifically indicate that the client's self-esteem has improved or that she has accepted her diagnosis. Saying that her friends will probably desert her if she tells them about the illness indicates that the adolescent still needs to work on her self-esteem and her feelings about the disease. Asking her friends what they think of someone with a lifelong illness would not indicate that the nurse's interventions targeted toward improving self-esteem have been successful. Rather, this statement demonstrates the adolescent's uncertainty about herself.

Painless chancre lesions are associated with which systemic disease? a) Urticaria b) Psoriasis c) Syphilis d) Kaposi's sarcoma

Syphilis Explanation: Syphilis is manifested by a painless chancre lesion. Psoriasis is exhibited by plaques with scales. Kaposi's sarcomas are cutaneous lesions blue-red or dark brown in color. Urticaria is wheals or hives due to infection or allergic reactions. (less)

A female client comes to the clinic and tells the nurse, "I think I have another vaginal infection and I also have some wartlike lesions on my vagina. This is happening quite often."What should the nurse consult with the physician regarding? a) Using a medicated douche in order to keep the vaginal pH normal b) Instructing the client to wear cotton underwear c) Testing the client for the presence of HIV d) Having the client abstain from sexual activity for 6 weeks while the medication is working

Testing the client for the presence of HIV Explanation: Abnormal results of Papanicolaou tests, genital warts, pelvic inflammatory disease, and persistent vaginitis also may correlate with HIV infection. Wearing cotton underwear can help with the prevention of candidiasis but does not address the recurrent vaginal infection that may not be caused by a fungus. Abstaining from sexual intercourse does not address the recurrent vaginal infection. A medicated douche can alter the normal flora of the vaginal wall

A 21-year-old female is talking with a nurse at her gynecologist's office. She is scheduled to have her first Pap test and she asks the nurse why she needs to have this test done. Which of the following is the nurse's best response? a) The test is used to detect early cancer of the cervix and to determine estrogen activity as it relates to menopause or endocrine abnormalities. b) The test is used to detect early cancer of the cervix and to determine progesterone activity as it relates to menopause or endocrine abnormalities. c) The test is used to detect early cancer of the ovaries and to determine progesterone activity as it relates to menopause or endocrine abnormalities. d) The test is used to detect early cancer of the vagina and to determine estrogen activity as it relates to menopause or endocrine abnormalities.

The test is used to detect early cancer of the cervix and to determine estrogen activity as it relates to menopause or endocrine abnormalities. Correct Explanation: This correctly describes the purpose of a Pap test. A Pap test is performed to detect early cancer of the cervix and to determine estrogen activity as it relates to menopause or endocrine abnormalities.

When performing an assessment, the nurse should focus on the developmental stage for which of the following clients? a) Adolescent b) Toddler c) Middle-aged adult d) Young adult

Toddler Correct Explanation: Nursing assessments vary according to the client's developmental needs. When assessing an infant, toddler, or child, special attention is given to physiological and psychosocial aspects of growth and development to identify client problems. It is not as important to focus assessment on the developmental stages in the other age groups

A nurse is implementing appropriate infection control precautions for a client who is positive for human immunodeficiency virus (HIV). The nurse demonstrates a need for a review of transmission routes by identifying which body fluid as a means of transmission? a) Urine b) Semen c) Blood d) Breast milk

Urine Correct Explanation: HIV is transmitted in body fluids that contain free virions and infected CD4+ T cells. These fluids include blood, seminal fluid, vaginal secretions, amniotic fluid, and breast milk. Urine is not a body fluid responsible for HIV transmission

Which instructions about breastfeeding should the nurse include when counseling the client? a) Wrap breasts with ace bandages. b) Use a water-based lubricant when having intercourse. c) Apply ice packs to breasts while nursing. d) Avoid fruit juices with acid.

Use a water-based lubricant when having intercourse. Explanation: Vaginal tissue can be atrophic while breastfeeding, so the use of a water-based lubricant can help with discomfort and lubrication during intercourse. Ice is applied to decrease breast milk supply and ace bandages are only used to dry up breast milk. Fruit juice is a health drink while breastfeeding

Parents of a 2-year-old child with chronic otitis media are concerned that the disorder has affected their child's hearing. Which behavior suggests that the child has a hearing impairment? a) Using gestures to express desires b) Playing alongside rather than interacting with peers c) Stuttering d) Babbling continuously

Using gestures to express desires Correct Explanation: Using gestures instead of verbal communication to express desires — especially in a child older than age 15 months — may indicate a hearing or communication impairment. Stuttering is normal in children ages 2 to 4, especially boys. Continuous babbling is a normal phase of speech development in young children. In fact, its absence, not presence, would be cause for concern. Parallel play — playing alongside peers without interacting — is typical of toddlers. However, in an older child, difficulty interacting with peers or avoiding social situations may indicate a hearing deficit.

The nurse is assessing the sexual health of a 20-year-old female client. During the interview, the client says, "I feel that my vaginal opening constricts whenever I am about to have intercourse. I seem to have no control over it." Which of the following would the nurse use when documenting the client's complaint? a) Celibacy b) Vaginismus c) Dyspareunia d) Orgasmic dysfunction

Vaginismus Correct Explanation: The client is experiencing an involuntary contraction of the muscles surrounding the vaginal orifice; this should be documented as vaginismus. Dyspareunia is painful intercourse. Difficulty achieving orgasm is documented as orgasmic dysfunction. Abstention from sexual intercourse is documented as celibacy

A nurse needs to administer a prescribed injection to a toddler. Which of the following injection sites is most suitable for the client? a) Ventrogluteal site b) Dorsogluteal site c) Deltoid site d) Vastus lateralis site

Vastus lateralis site Correct Explanation: The vastus lateralis site is the most desirable site for administering injections to infants and small children and clients who are thin or debilitated with poorly developed gluteal muscles. The dorsogluteal site is avoided in clients younger than 3 years because their gluteus maximus muscle is not sufficiently developed; whereas, the ventrogluteal site is safe for children. The deltoid site is the least-used intramuscular injection site because it is a smaller muscle than the others. It is used only for adults because the muscle is not sufficiently developed in infants and children.

A nurse is assisting a clinician to perform a complete pelvic examination on a female client. Which of the following should be done when performing a speculum examination? Select all that apply. a) Warm the speculum before insertion. b) Withdraw the speculum and obtain smear for culture. c) Expand the speculum after insertion into the vagina. d) View the vaginal walls as the speculum is withdrawn. e) Palpate the cervix, ovaries, and uterus with the speculum.

View the vaginal walls as the speculum is withdrawn. • Expand the speculum after insertion into the vagina. • Warm the speculum before insertion. Correct Explanation: When performing a speculum examination, the speculum should be warmed before insertion. After insertion in the vagina, the speculum is expanded for viewing. The vaginal walls are viewed as the speculum is withdrawn. Samples and smears for culture are taken while the speculum is in place, not after it is withdrawn. In bimanual palpation, the clinician places the index and middle fingers of one hand into the vagina while placing the other hand on the lower abdomen. The cervix, ovaries, and uterus are palpated by this method, not using a speculum.

A nurse is teaching the family of an 8-year-old boy with acute lymphocytic leukemia about appropriate activities. Which recommendation should the nurse make? a) restriction from participating in athletic activities b) home schooling c) avoiding trips to the shopping mall d) being treated as "normal" as much as possible

being treated as "normal" as much as possible Correct Explanation: Any child with a chronic illness should be treated as normally as possible. Unless the child has severe bone marrow depression, he should be allowed to go to school with others and can go to the mall. If the child is in remission, athletic activities are allowed

The nurse is evaluating an infant for auditory ability. What is the expected response in an infant with normal hearing? a) blinking and stopping body movements when sound is introduced b) absence of squealing by age 4 months c) saying "da-da" by age 5 months d) evidence of shy and withdrawn behaviors

blinking and stopping body movements when sound is introduced Explanation: In response to hearing a noise, normally hearing infants blink or startle and stop body movements. Shy and withdrawn behaviors are characteristic of older children with hearing impairment. Squealing occurs in 90% of infants by age 4 months. Most infants can say "da-da" by age 9 months.

To assess a 9-year-old's social development, the nurse asks the parent if the child: a) enjoys active play. b) has a best friend. c) thinks independently. d) is able to organize and plan.

has a best friend. Explanation: During the school-age years children learn to socialize with children of the same age. The "best friend" stage, which occurs around 9 or 10 years of age, is important in providing a foundation for self-esteem and later relationships. Thinking independently, organizing, and planning are cognitive skills. Active play relates to motor skills

A nurse is assessing the growth and development of a 10-year-old. What is the expected behavior of this child? a) enjoys physical demonstrations of affection b) is uncooperative in play and school c) has a strong sense of justice and fair play d) is selfish and insensitive to the welfare of others

has a strong sense of justice and fair play Correct Explanation: School-age children are concerned about justice and fair play. They become upset when they think someone is not playing fair. Physical affection makes them embarrassed and uncomfortable. They are concerned about others and are cooperative in play and school.

While caring for a pregnant adolescent, a nurse should develop a care plan that incorporates the adolescent's: a) healthy eating habits. b) level of emotional maturity. c) age of menarche. d) family and home life.

level of emotional maturity. Correct Explanation: When assessing an adolescent initially, the nurse should try to determine the adolescent's level of emotional maturity. This information is used as the basis for the nursing care plan. Age of menarche, family and home life, and healthy eating habits, though important, aren't as significant as determining the emotional maturity of the adolescent.

After surgery to correct a tetralogy of Fallot, the child's parents express concern to the nurse that their 4-year-old child wants to be held more frequently than usual. The nurse recommends: a) encouraging the behavior. b) play therapy. c) introducing a new skill. d) having the volunteer hold the child.

play therapy. Correct Explanation: The child is exhibiting regression. During periods of stress, children frequently revert to behaviors that were comforting in earlier developmental stages; play therapy is one way to help the child cope with the stress. Teaching a new skill most likely would add more stress. Parents should be instructed to praise positive behaviors and ignore regressive behaviors rather than calling attention to them through encouragement or discouragement. Having someone else hold the child does not encourage coping with the stress or promoting appropriate development.

A 7-year-old child is admitted to the hospital with acute rheumatic fever. During the acute phase of the illness, which diversional activity would the nurse most discourage? a) playing with a doll with the nurse b) playing checkers with a roommate c) reading a book with the father d) watching the television with a sibling

playing checkers with a roommate Correct Explanation: School-aged children enjoy board games and are commonly intense about following rules. Their play can become emotional. Adequate rest is of utmost importance during the acute stage of rheumatic fever. Therefore, playing a game with another child probably would be too strenuous. Such diversional activities as reading a book, playing with a doll, and watching television would be more satisfactory.

A client is diagnosed with genital herpes, (herpes simplex virus type 2, or HSV-2). The nurse should instruct the client that: a) there are no effective drug therapies to manage herpes symptoms. b) reducing stressful life events may decrease the incidence of herpetic outbreaks. c) herpes is transmitted to partners only when lesions are weeping. d) using occlusive ointments may decrease the pain from the lesions.

reducing stressful life events may decrease the incidence of herpetic outbreaks. Explanation: Managing stressful life events can decrease the incidence of outbreaks of HSV-2. Occlusive ointments should not be applied. Antiviral therapies will not cure herpes, but they can manage symptoms and decrease the incidence of outbreaks. Clients with HSV-2 should use condoms to prevent HSV transmission. Cells can be shed at other times, not only when the vesicles are weeping

Which of the following is appropriate language development for an 8-month-old? The child should be: a) saying "ball" when parents point to a ball. b) saying "dada" and "mama" specifically ("dada" to father and "mama" to mother). c) saying "dada" and "mama" nonspecifically. d) saying three other words besides "mama" and "dada."

saying "dada" and "mama" nonspecifically. Correct Explanation: It is important for the nurse to assist parents in assessing speech development in their child so that developmental delays can be identified early. According to the Denver Developmental Screening Examination, at 8 months of age, the child should say "mama" and "dada" nonspecifically and imitate speech sounds. Children cannot say "dada" or "mama" specifically or use more than three words until they are about 12 months of age. A child cannot respond to specific commands or point to objects when requested until about 17 months of age

A 28-year-old male is diagnosed with acute epididymitis. The nurse should assess the client for: a) foul-smelling urine. b) burning and pain on urination. c) foul-smelling ejaculate. d) severe tenderness and swelling in the scrotum.

severe tenderness and swelling in the scrotum. Correct Explanation: Epididymitis causes acute tenderness and pronounced swelling of the scrotum. Gradual onset of unilateral scrotal pain, urethral discharge, and fever are other key signs. Epididymitis is occasionally, but not routinely, associated with urinary tract infection. Burning and pain on urination and foul-smelling ejaculate or urine are not classic symptoms of epididymitis.

A nurse is planning care for a 7-year-old who is hospitalized for a hernia repair. The nurse should assess the client for which fear common in this age groupt? a) separation from parents b) injury and pain c) trying something new d) opposite-sex relationships

trying something new Explanation: Trying something new is usually frightening for a 7-year-old. Separation anxiety is the most common fear between the ages of 5 months and 5 years of age. Injury and pain are a common fear of the preschool child. Fear of the opposite sex is common during adolescence.

Whichis an initial clinical manifestation of gonorrhea in men? a) impotence b) urethral discharge c) scrotal pain d) penile lesion

urethral discharge Correct Explanation: Urethritis is usually the initial clinical manifestation of gonorrhea in men. The symptoms include a profuse, purulent discharge and dysuria. Complications are uncommon, but they include prostatitis and sterility. Impotence, scrotal pain, and penile lesions are not associated with gonorrhea.

To promote comfort and optimal respiratory expansion for a client with chronic obstructive pulmonary disease during sexual intimacy, the nurse can suggest that the couple: a) use a foam mattress. b) use pillows to raise the affected partner's head and upper torso. c) have the affected partner assume a dependent position. d) limit the duration of the sexual activity.

use pillows to raise the affected partner's head and upper torso. Explanation: Raising the upper torso for the affected partner facilitates respiratory function. The use of a foam mattress is not necessary and will not help promote respiratory expansion. A dependent position may compromise respiratory expansion, even though energy may be conserved. Duration of sexual activity is not necessarily related to exertion

The parent of a 9-month-old expressed concern that the baby "is developing slowly." The nurse is concerned about a developmental delay when finding the baby is unable to accomplish which skill? a) drinking from a cup with little spilling b) building a tower of two cubes c) vocalizing single syllables d) standing alone

vocalizing single syllables Correct Explanation: Normally, a 9-month-old infant should have been voicing single syllables since 6 months of age. Absence of this finding would be a cause for concern. An infant usually is able to stand alone at about 10 months of age. An infant usually is able to build a tower of two cubes at about 15 months of age. An infant usually is able to drink from a cup with little spilling at about 15 months of age.

A nurse is assessing neonates in a hospital nursery. Which neonates are exhibiting normal characteristics? (Select all that apply.) a) The neonate inherits a transient immunity from infections from the mother. b) A neonate's senses are not developed enough to feel pain from a heel-stick. c) A neonate eliminates urine and stool. d) A neonate displays the Moro and stepping reflex. e) A neonate's body temperature responds slowly to environmental temperature. f) A neonate exhibits both an active crying state and a quiet alert state.

• A neonate displays the Moro and stepping reflex. • A neonate eliminates urine and stool. • A neonate exhibits both an active crying state and a quiet alert state. • The neonate inherits a transient immunity from infections from the mother. Explanation: The neonate would exhibit reflexes to include the Moro reflex and the stepping reflex. A neonate does eliminate urine and stool. The neonate exhibits both an active crying state and a quiet alert state. The neonate inherits a transient immunity from infections from the mother. The neonate's body temperature responds quickly to environmental temperature. A neonate's senses are developed enough to feel pain from a heel-stick.

A 15-year-old girl visits the neighborhood clinic seeking information on "how to keep from getting pregnant." What should the nurse say in response to her request? a) "What would you like to know?" b) "Can you tell me if you've told your parents you're having sex?" c) "Let's discuss what your friends are doing to keep from getting pregnant." d) "Can you tell me about the precautions you're taking now?"

"Can you tell me about the precautions you're taking now?" Correct Explanation: An attitude that requests only the information the girl is willing to give is nonthreatening and nonjudgmental. This may enhance the girl's willingness to talk about her experiences, thus enabling the nurse to better assess her needs. Asking what the girl would like to know assumes the girl knows what information she needs. The precautions her friends are taking are irrelevant at this time. Referencing the girl's parents may make her defensive and fearful

A 23 year old female presents with the following symptoms: bleeding, feeling sad, headaches, increased acne, and recent weight changes. The nurse knows that the following need to be reviewed with the patient. a) Use of oral contraceptives in the past month b) Hormone imbalance caused by pregnancy c) Intrauterine device(IUD) placement within past 3 months d) Pregnancy related risk factors

Intrauterine device(IUD) placement within past 3 months Explanation: With intrauterine devices the clinician should be aware that adverse side effects peak at 3 months of use and reduce in frequency after that. The most common side effects include bleeding, depression, headache, acne, and weight changes. While some of the symptoms are found as side effects of oral contraceptive use and as signs of pregnancy the age of the client and cluster of symptoms should alert the nurse to assess for the presence of an IUD

Which observation by the nurse should suggest that a 15-month-old toddler has been abused? a) The child sucks his thumb. b) The child appears happy when personnel work with him. c) The child plays alongside others contentedly. d) The child is underdeveloped for his age.

The child is underdeveloped for his age. Correct Explanation: An almost universal finding in descriptions of abused children is underdevelopment for age. This may be reflected in small physical size or in poor psychosocial development. The child should be evaluated further until a plausible diagnosis can be established. A child who appears happy when personnel work with him is exhibiting normal behavior. Children who are abused often are suspicious of others, especially adults. A child who plays alongside others is exhibiting normal behavior, that of parallel play. A child who sucks his thumb contentedly is also exhibiting normal behavior.

Which of the following is not a definition of erectile dysfunction? a) The inability to achieve or maintain an erection that is sufficiently rigid for sexual activity b) The inability to achieve an erection after sexual activity c) The inability to sustain erection for a satisfactory period of time d) The inability to achieve an erection

The inability to achieve an erection after sexual activity Explanation: The ability to achieve an erection after sexual activity varies and depends on the man's age, health, and sexual excitement. This statement is included in the definition of erectile dysfunction. Additionally, there must be multiple or persistent incidences of failed erection for the disorder to be considered pathologic.

A right orchiectomy is performed on a client with a testicular malignancy. The client expresses concerns regarding his sexuality. The nurse should base the response on the knowledge that the client: a) should retain normal sexual drive and function. b) will be impotent. c) is not a candidate for sperm banking. d) will have a change in secondary sexual characteristics.

should retain normal sexual drive and function. Correct Explanation: Unilateral orchiectomy alone does not result in impotence if the other testis is normal. The other testis should produce enough testosterone to maintain normal sexual drive, functioning, and characteristics. Sperm banking before treatment is commonly recommended because radiation or chemotherapy can affect fertility

A young man with early-stage testicular cancer is scheduled for a unilateral orchiectomy. The client confides to the nurse that he is concerned about what effects the surgery will have on his sexual performance. Which response by the nurse provides accurate information about sexual performance after an orchiectomy? a) "We will refer you to a sex therapist because you will probably notice erectile dysfunction." b) "Most impotence resolves in a couple of months." c) "Because your surgery does not involve other organs or tissues, you will likely not notice much change in your sexual performance." d) "You could have early ejaculation with this type of surgery."

"Because your surgery does not involve other organs or tissues, you will likely not notice much change in your sexual performance." Correct Explanation: Although there may not be a big change in sexual function with a unilateral orchiectomy, the loss of a gonad and testosterone may result in decreased libido and sterility. Sperm banking may be an option worth exploring if the number and motility of the sperm are adequate. Remember, the population most affected by testicular cancer is generally young men aged 15 to 34, and in this crucial stage of life, sexual anxieties may be a large concern

A client has a cerclage placed at 16 weeks' gestation. She has had no contractions and her cervix is dilated 2 cm. The nurse is preparing the client for discharge. Which statement by the client should indicate to the nurse that the client needs further instruction? a) "I will need more frequent prenatal visits." b) "I can have sex again in about 2 weeks." c) "I can have nothing in my vagina until I am at term." d) "I should call if I am leaking fluid or have bleeding or contractions."

"I can have sex again in about 2 weeks." Explanation: Intercourse commonly stimulates uterine contractions. The prostaglandins found in semen can also initiate contractions. After placement of a cerclage for advanced dilation and contractions, the client is considered at high risk for preterm birth and should be seen by her health care provider (HCP) more frequently. The client should call the HCP immediately if she sees signs of complications, such as leaking fluid (rupture of membranes), vaginal bleeding, and contractions (particularly with a cerclage in place). Anything in the vagina may initiate contractions and the labor process.

As a first step in teaching a woman with a spinal cord injury and quadriplegia about her sexual health, the nurse assesses her understanding of her current sexual functioning. Which statement by the client indicates she understands her current ability? a) "I will not be able to have sexual intercourse until the urinary catheter is removed." b) "I should be able to participate in sexual activity, but I will be infertile." c) "I can participate in sexual activity but might not experience orgasm." d) "I cannot have sexual intercourse because it causes hypertension, but other sexual activity is okay."

"I can participate in sexual activity but might not experience orgasm." Correct Explanation: The woman with spinal cord injury can participate in sexual activity but might not experience orgasm. Cessation in the nerve pathway may occur in spinal cord injury, but this does not negate the client's mental and emotional needs to creatively participate with her partner in a sexual relationship and to reach orgasm. An indwelling urinary catheter may be left in place during intercourse and need not be removed because the indwelling urinary catheter is placed in the urethra, which is not the channel used for sexual intercourse. There are no contraindications, such as hypertension, to sexual activity in a woman with spinal cord injury. Sexual intercourse is allowed, and hypertension should be manageable. Because a spinal cord injury does not affect fertility, the client should have access to family planning information so that an unplanned pregnancy can be avoided.

A nurse is teaching a client with genital herpes. Which of the following would indicate to the nurse that the client's teaching was successful? a) "The disease is transmittable only when visible lesions are present." b) "As long as I am in a monogamous relationship, I do not have to use safer-sex practices." c) "I can use condoms made from animal skins." d) "I should inform my partners about the disease."

"I should inform my partners about the disease." Correct Explanation: Clients with genital herpes should inform their partners of the disease to help prevent transmission. Animal skin condom should be avoided because the products do not prevent virus transmission. The notion that genital herpes is only transmittable when visible lesions are present is false. Anyone not in a long-term, monogamous relationship, regardless of current health status, should follow safer-sex practices if having sex with an infected partner

The nurse is teaching a client with HIV how to use a male condom. The client demonstrates understanding of the information when he states which of the following? Select all that apply. a) "I need to hold the condom by the tip to squeeze out the air before putting it on." b) "I should unroll the condom all the way over my erect penis." c) "I can keep several condoms in my wallet in my back trouser pocket so they are readily available." d) "I can reuse a condom two or three times when I'm having sexual intercourse." e) "I should avoid using baby oil or petroleum jelly with a condom."

"I should unroll the condom all the way over my erect penis." • "I should avoid using baby oil or petroleum jelly with a condom." • "I need to hold the condom by the tip to squeeze out the air before putting it on." Correct Explanation: The client should apply a new male condom before any kind of sex and should use the condom only once. He also should use a new condom for sex in a different place, such as the anus and then in the vagina. The client should hold the condom by the tip to squeeze the air out and then unroll the condom all the way over the erect penis. Condoms should be stored in a cool, dry place, not the wallet or back pocket because the condom can break down. Products containing oil such as baby oil, skin lotions, or petroleum jelly should not be used because these products will cause the condom to break.

Cone biopsy is a term used to describe a procedure in which cervical tissue is removed as result of detection of abnormal cells. Which of the following statements made by the patient demonstrates that the patient undergoing a cone biopsy understands the discharge instructions? a) "I will use a sitz bath to relieve pain caused by the sutures." b) "I will avoid having sexual relations until I see the doctor again." c) "I require a repeat conization in 2 weeks after the edema subsides." d) "I will need to use a menstrual pad to capture the moisture as my cervix unfreezes."

"I will avoid having sexual relations until I see the doctor again." Correct Explanation: Cone biopsy is a procedure done to remove abnormal cervical tissue. The nurse or primary provider provides guidelines regarding postoperative sexual activity, bathing, and other activities. Because open tissue may be potentially exposed to HIV and other pathogens, the patient is cautioned to avoid intercourse until healing is complete and verified at follow-up. Routine repeat cone biopsy is not normally indicated. Perineorraphy refers to sutural repair of perineal lacerations; sutures are not required with a cone biopsy. Cryotherapy refers to destruction of tissue by freezing; no freezing is involved with a cone biopsy.

A client is scheduled to have in vitro fertilization (IVF) as an infertility treatment. Which client statement about IVF indicates that the client understands this procedure? a) "IVF involves bypassing the blocked or absent fallopian tubes." b) "Both ova and sperm are instilled into the open end of a fallopian tube." c) "The pregnancy rate with IVF is higher than that with gamete intrafallopian transfer." d) "IVF requires supplemental estrogen to enhance the implantation process."

"IVF involves bypassing the blocked or absent fallopian tubes." Explanation: The client's understanding of the procedure is demonstrated by the statement describing IVF as a technique that involves bypassing the blocked or absent fallopian tubes. The primary health care provider removes the ova by laparoscope- or ultrasound-guided transvaginal retrieval and mixes them with prepared sperm from the woman's partner or a donor. Two days later, up to four embryos are returned to the uterus to increase the likelihood of a successful pregnancy. Supplemental progesterone, not estrogen, is given to enhance the implantation process. Gamete intrafallopian transfer (GIFT) and tubal embryo transfer have a higher pregnancy rate than IVF. However, these procedures cannot be used for clients who have blocked or absent fallopian tubes because the fertilized ova are placed into the fallopian tubes, subsequently entering the uterus naturally for implantation. In IVF, fertilization of the ova by the sperm occurs outside the client's body. In GIFT, both ova and sperm are implanted into the fallopian tubes and allowed to fertilize within the woman's body.

A gerontology nurse is discussing sexual dysfunction with an elderly male client. The nurse sees that learning has occurred when the client states which of the following? a) "I need to be able to walk a mile in order for sexual intercourse to not be dangerous." b) "Malnutrition may cause impotence." c) "Antihypertensive medication will not affect sexual function." d) "Erectile dysfunction, caused by diabetes, has no treatment."

"Malnutrition may cause impotence." Correct Explanation: Factors that contribute to sexual dysfunction in the older adult are as follows: use of medications, especially antihypertensives; age-related metabolic disorders such as anemia, diabetes, malnutrition, and fatigue may cause impotence; sexual intercourse and similar forms of sexual expression are not considered dangerous for anyone able to walk around a room; and erectile dysfunction has treatments such as a penile prosthesis or pharmacological management

A teenage female client tells the nurse that she is having intercourse with her boyfriend. The client asks the nurse about methods to prevent pregnancy. Which of the following statements by the nurse would be most accurate? a) "Barrier methods include the vaginal sponge used in combination with an antibacterial agent." b) "Coitus interruptus will prevent pregnancies." c) "Periodic abstinence involves not having any sex with a partner." d) "Many unintended pregnancies result from the use of less effective methods of contraception."

"Many unintended pregnancies result from the use of less effective methods of contraception." Explanation: Unintended pregnancy remains a significant women's health issue in the United States. Many unintended pregnancies result from the use of less effective methods of contraception, such as condoms, spermicide, or barrier methods. Continuous abstinence involves not having sex with a partner at all. In periodic abstinence, women chart their periods of fertility and abstain from vaginal intercourse during these periods. Barrier methods include the vaginal sponge used in combination with a spermicidal agent. Coitus interruptus, the withdrawal of the penis from the vagina before ejaculation, does not prevent all pregnancies. Pre-ejaculate can contain enough sperm to cause a pregnancy, and pre-ejaculate or semen may spill onto the vulva

A 52-year-old client is scheduled for a total abdominal hysterectomy for cervical cancer. When discussing the potential impact of this procedure on the client's sexuality, the nurse should respond by saying: a) "All women experience sexual problems with this surgical procedure. Do you have any questions?" b) "When can I schedule an appointment with you and your partner to discuss any issues either of you may have regarding sexuality?" c) "Do you anticipate any problems with sex related to your scheduled hysterectomy?" d) "Most women have concerns about their sexuality after this type of surgery. Do you have any concerns or questions?"

"Most women have concerns about their sexuality after this type of surgery. Do you have any concerns or questions?" Correct Explanation: This question introduces some basic information and allows for support for the client who may be experiencing some sexuality concerns. Not all women experience sexual problems after undergoing a hysterectomy. Assuming that the client will want to schedule an appointment with her partner is inappropriate and may embarrass her. Simply asking the client whether she expects to have problems with sex is too abrupt and does not provide any information

Students in a health class are discussing birth control and prevention of sexually transmitted disease. The school nurse would know that teaching has been effective if the students state which of the following? a) "Safe sex means preventing pregnancy through use of birth control." b) "The intrauterine device is the most effective way to prevent pregnancy." c) "Responsible sex involves using condoms and spermicides for protection and birth control." d) "The rhythm method means not having sex just before menstruation."

"Responsible sex involves using condoms and spermicides for protection and birth control." Correct Explanation: This comment indicates an understanding of ways to lessen the incidence of sexually transmitted illnesses by condom use. It also indicates that use of a spermicide and condom will help to prevent unwanted pregnancies. The other choices are not accurate examples of safer sex.

The nurse in a cardiac clinic is taking vital signs of a 58-year-old man who is 3 months status post myocardial infarction (MI). While the physician is seeing the client, the client's spouse approaches the nurse and asks about sexual activity. "We are too afraid he will have another heart attack, so we just don't have sex anymore." The nurse's best response is which of the following? a) "The physiologic demands are greatest during orgasm and are equivalent to walking 3 to 4 miles per hour on a treadmill." b) "It is usually better to just give up sex after a heart attack." c) "The medications will prevent your husband from having an erection." d) "Having an orgasm is very strenuous and your husband must be in excellent physical shape before attempting it."

"The physiologic demands are greatest during orgasm and are equivalent to walking 3 to 4 miles per hour on a treadmill." Correct Explanation: The physiologic demands are greatest during orgasm. The level of activity is equivalent to walking 3 to 4 miles per hour on a treadmill. Erectile dysfunction may be a side effect of beta-blockers, but other medications may be substituted

An 56-year-old menopausal client comes to the clinic reporting painful intercourse. Her pelvic examination reveals vaginal dryness. Which suggestion would be most appropriate? a) "Try using a water-soluble lubricant before intercourse." b) "Drink at least 6 to 8 glasses of water daily." c) "Practice your Kegel exercises several times each day." d) "Make sure you take your calcium supplement everyday."

"Try using a water-soluble lubricant before intercourse." Correct Explanation: Dyspareunia (painful intercourse) in a menopausal woman is most likely from vaginal dryness. Use of a water-soluble lubricant can be helpful in adding moisture to the area and relieving the pain of intercourse. Kegel exercises would be helpful to combat problems related to bladder control. Calcium supplements would be helpful to prevent osteoporosis. Drinking 6 to 8 glasses of water daily helps reduce the woman's risk for urinary tract infections.

A 36-year-old female is scheduled to receive external radiation therapy and a cesium implant for cancer of the cervix and is asking about the effects of the radiation on sexual relations. The nurse should inform the client about which potential effects of radiation therapy on sexuality? a) "You can have sexual intercourse while the implant is in place." b) "You may notice some vaginal dryness after treatment is completed." c) "You will continue to have normal menstrual periods during treatment." d) "You may notice some vaginal relaxation after treatment is completed."

"You may notice some vaginal dryness after treatment is completed." Correct Explanation: Radiation fields that include the ovaries usually result in premature menopause. Vaginal dryness will occur without estrogen replacement. There should be no sexual intercourse while the implant is in place. Cesium is a radioactive isotope used for therapeutic irradiation of cancerous tissue. There is no documentation to support vaginal relaxation after treatment. Because the client will have premature menopause, she will not have normal menstrual periods

The mother of an older infant reports stopping the prescribed iron supplements after 2 weeks of treatment. Which response by the nurse is most appropriate? a) "Bring the child in so that we can retest him." b) "No more medication is needed at this time." c) "Let us start a diet that is high in iron." d) "You need to continue the iron for several more weeks."

"You need to continue the iron for several more weeks." Correct Explanation: Typically, iron supplements are needed for at least 1 month. By the end of this time, there should be a significant rise in the hemoglobin and hematocrit. Therefore, the mother needs to continue the iron supplements for several more weeks. Testing the child after only 2 weeks of treatment may not be beneficial. A significant rise in hemoglobin and hematocrit usually requires approximately 1 month of therapy. An iron-rich diet should have been started when the diagnosis was made and continued for at least the duration of iron supplement therapy

Which intervention would be most appropriate to institute when a school-age child with burns becomes angry and combative when it is time to change the dressings and apply mafenide acetate? a) Ensure parental support during the dressing changes. b) Allow the child to assist in removing the dressings and applying the cream. c) Give the child permission to cry during the procedure. d) Allow the child to schedule the time for dressing changes.

Allow the child to assist in removing the dressings and applying the cream. Correct Explanation: Expressions of anger and combativeness are often the result of loss of control and a feeling of powerlessness. Some control over the situation is regained by allowing the child to participate in care. Although having parental support during the dressing changes may be helpful, this action does nothing to allow the child control. Giving the child permission to cry may help with verbalizing feelings, but doing so does nothing to provide the child with control over the situation. Although allowing the child to determine the time for dressing changes may provide a sense of control over the situation, doing so is inappropriate because the dressing changes need to be performed as prescribed to ensure effectiveness and healing.

When performing a physical examination on a neonate, the nurse notes low-set ears. What action should the nurse perform next? a) Call the pediatrician for an immediate evaluation of the infant. b) Note the findings in the medical record. c) Order an ultrasound of the head to determine if the brain is normal. d) Assess the neonate to determine if other apparent abnormalities are present.

Assess the neonate to determine if other apparent abnormalities are present. Explanation: Although low-set ears are an abnormal finding, the presence of this abnormality by itself isn't cause for immediate concern. The nurse should continue to assess the neonate to determine if other abnormalities are present. It's appropriate to note the abnormality in the medical record; however, it's even more important to continue the assessment. It's outside the scope of nursing practice to order a diagnostic test, such as an ultrasound, and there's no indication for this test

A pediatric nurse practitioner is assessing the development of a child. The nurse's assessment of development will focus on which of the following? a) A static process that occurs during childhood b) Changes in the body's cell structure, function, and complexity c) Changes in thoughts, feelings, and behaviors d) Increases in the child's body size

Changes in thoughts, feelings, and behaviors Explanation: Development is an orderly pattern of change in structure, thoughts, feelings, or behaviors resulting from maturation, experiences, and learning. Development is a dynamic and continuous process as one proceeds through life, characterized by a series of ascents, plateaus, and declines. Growth, on the other hand, is an increase in body size, or changes in body cell structure, function, and complexity.

The nurse prepares to complete an exam on a 25 year old female. The client presents to the community health clinic with complaints of difficulty with urination. The nurse reviews the chart and sees that the client is originally from Africa. The nurse suspects which of the following? a) Complications from an untreated sexually transmitted infection (STI) b) A diagnosis of interstitial cystitis c) Complications from female genital mutilation (FGM) d) An acute urinary tract infection (UTI)

Complications from female genital mutilation (FGM) Correct Explanation: Female genital mutilation (FGM) includes procedures that intentionally alter or cause injury to the female genital organs for non-medical reasons. It is a procedure that has no health benefits for girls and women and can cause severe bleeding, problems urinating, and later cysts, infections, infertility as well as complications in childbirth and increased risk of newborn deaths. The nurse may rule out UTI via urinalysis however, the nurse would want to rule out the presence of FGM in order to make sure the client receives appropriate long-term care. Nothing in the data provides the nurse with an indication of an untreated STI or interstitial cystitis.

A female patient experienced a severe reduction in body fat by extreme caloric restriction and exercise. Which of the following female reproductive abnormalities could she acquire as a result of the reduction in body fat? a) Polycystic ovarian syndrome b) Small breasts c) Delay or cessation of menstruation d) Higher risk of fibrocystic breast disease

Delay or cessation of menstruation Correct Explanation: She could acquire reproductive abnormalities, such as delay or cessation of menstruation. A cause of infertility in women is polycystic ovarian syndrome a multi-endocrine disorder linked to insulin resistance. High caffeine consumptions may also put a patient at risk of fibrocystic breast disease. (

A female patient is scheduled to undergo a Papanicolaou test. The nurse should advise the patient of which of the following preparations? a) Avoid consuming food that contains iodine 48 hours prior to the procedure b) Do not douche prior to procedure c) Make sure procedure is scheduled during menstruation d) Avoid intercourse for two days prior to the procedure

Do not douche prior to procedure Explanation: To avoid washing away cellular material, the patient should be instructed not to douche before having a Pap smear taken. A Pap smear should be performed when a patient is not menstruating because blood usually interferes with interpretation. Consuming food that contains iodine and intercourse is not contraindicated for a Papanicolaou test.

The parent of a preschooler reports that the child creates a scene every night at bedtime. What is the best course of action? a) Encourage active play before bedtime. b) Allow the child to stay up later one or two nights a week. c) Establish a set bedtime and follow a routine. d) Give the child a cookie if bedtime is pleasant.

Establish a set bedtime and follow a routine. Correct Explanation: Bedtime is often a problem with preschoolers. Recommendations for reducing conflicts at bedtime include establishing a set bedtime, having a dependable routine, such as story reading, and conveying the expectation that the child will comply. Allowing the child to stay up late one or two nights interferes with establishing the needed bedtime rituals. Excitement, such as active play, just before bedtime should be avoided because it stimulates the child, making it difficult for the child to calm down and prepare for sleep. Using food such as a cookie as a reward if bedtime is pleasant should be avoided because it places too much importance on food. Other rewards, such as stickers, could be used as an alternative

A female client tells the nurse that vaginal intercourse with her partner is not as exciting as it used to be. Which of the following could the nurse teach her? a) Use of the BETTER model b) Masturbation c) Voyeurism d) Use of Kegel exercises

Use of Kegel exercises Correct Explanation: Some women who practice Kegel exercises, which promote good vaginal tone, have found their sexual satisfaction is improved. The BETTER model is a tool used in conducting sexual assessments. Voyeurism and masturbation are techniques of self- stimulation that would not involve the client's partner.

Which of the following is the most common sexually transmitted infection (STI) among young, sexually active people? a) Chlamydia b) Human papillomavirus (HPV) c) Gonorrhea d) Trichomoniasis

Human papillomavirus (HPV) Explanation: HPV infection is the most common STI among young, sexually active people. Millions of Americans are infected with HPV, many unaware that they carry the virus.

The critical care nurse is caring for clients in an emergency department. When caring for a variety of clients, when is the presence of a third heart sound normal? a) In clients with heart valve replacement b) In clients with an indwelling pacemaker c) In pediatric clients d) In geriatric clients

In pediatric clients Correct Explanation: When caring for a variety of clients, it is important to consider that a third heart sound is normal in children. In adults, a third heart sound may signify heart failure. There is no correlation between third heart sounds with heart valve replacement and an indwelling pacemaker.

Adolescent sexual development occurs in three stages: prepubescence, pubescence, and postpubescence. Characteristics common to the phase of pubescence include which of the following? a) Appearance of breast buds in females and increase in length and circumference of penis in males b) Increase in the amount, pigmentation, and curling of pubic hair and growth spurt in height and weight c) Completion of sexual growth and development and fertility d) Slight enlargement of sex organs and appearance of downy pubic hair

Increase in the amount, pigmentation, and curling of pubic hair and growth spurt in height and weight Explanation: Characteristics common to the pubescence phase include an increase in the amount, pigmentation, and curling of pubic hair and growth spurt in height and weight. Axillary hair appears and no longer has a downy appearance. Sex organs continue to enlarge, and ovulation and spermatogenesis begins. Completion of sexual growth and development and fertility occur during the postpubescence phase. Appearance of breast buds in females and increase in length and circumference of penis in males and progressive enlargement of sex organs and appearance of downy pubic hair occur during the prepubescence phase.

According to Erikson's psychosocial theory of development, an 8-year-old child would be in which stage? a) Identity versus role confusion b) Industry versus inferiority c) Trust versus mistrust d) Initiative versus guilt

Industry versus inferiority Explanation: In middle childhood, the 6- to 12-year-old child is mastering the task of industry versus inferiority. The trust versus mistrust task is in infancy (birth to 1 year). In early childhood, the 1- to 3-year-old child is in the stage of initiative versus guilt. Identity versus role confusion occurs during adolescence.

At the health clinic, a sexually active 15-year-old girl tells a nurse she's worried that her parents may find out about her sexual activity. "They would never approve," she says. The nurse should formulate which nursing diagnosis? a) Fear related to boyfriend's expectations b) Impaired social interaction related to boyfriend's expectations c) Delayed growth and development related to sexual activity d) Ineffective sexuality patterns related to parent's expectations

Ineffective sexuality patterns related to parent's expectations Correct Explanation: This girl is expressing concerns about the conflict between her parent's expectations and her own desires. Sexual activity is a normal experimental pattern for many adolescents, but she verbalizes parental expectations against this behavior. No evidence suggests she's having a conflict with her boyfriend, delayed growth, or problems with social interactions.

A nurse is working in a school health program that provides care to elementary school-aged children. The nurse would focus teaching on which area as being most problematic? a) Sexually transmitted infections b) Injuries c) Substance abuse d) Eating disorders

Injuries Correct Explanation: Injuries are a leading health problem for childen in elementary school. Therefore, the nurse would focus teaching on this area. Substance abuse, eating disorders, and sexually transmitted infections are leading problems associated with students in high school.

A student nurse is enrolled in a growth and development class. Which of the following Freudian stages should the student nurse recognize in a child with an increase in gender difference awareness? a) Oral Stage b) Phallic Stage c) Anal Stage d) Latency Stage

Phallic Stage Correct Explanation: The Phallic Stage is the stage in which a child develops an increased awareness of gender differences. The Oral Stage is when the infant uses his or her mouth as a major source of satisfaction. The Anal Stage is when toilet training is taking place with the child. The Latency Stage involves identification with the same sex parent

As a nursing student, you are visiting a day care to observe growth and development in action. You do screening assessments on preschoolers. They are learning to feed, dress, and toilet themselves, as well as expanding their motor and verbal abilities. According to Piaget, in what development stage are they? a) Concrete operational b) Preoperational c) Autonomy versus shame and doubt d) Sensorimotor

Preoperational Explanation: Piaget's preoperational stage is characterized by the beginning use of symbols, through increased language skills and pictures, to represent the preschooler's world. Sensorimotor is Piaget's birth to 24-month stage. Concrete operational is Piaget's 7- to 11-year-old stage. Autonomy versus shame and doubt is Erikson's theory

The nurse is working in the labor and child birth unit when a mother with active herpes simplex virus-Type 2 (HSV-2) appears in active labor. Which adjustment in the plan of care is anticipated? a) Place an antibacterial ointment on the mother's lesions b) Prepare the mother for a cesarean section c) Complete a full assessment of the newborn on birth d) Administer an intravenous antibiotic to the mom while in labor

Prepare the mother for a cesarean section Correct Explanation: The nurse is most accurate to prepare for a cesarean section since the mother has an active lesion and does not want to transmit the virus to the newborn. Antibiotic therapy, at this time, does not prevent the transmission of the infection. A full assessment is always completed on the newborn and is not an adjustment in the plan of care. Antibacterial ointment is not placed on the mother's lesions

When developing a care plan for a hospitalized child, the nurse knows that children in which age-group are most likely to view illness as a punishment for misdeeds? a) School age b) Preschool age c) Adolescence d) Infancy

Preschool age Correct Explanation: Preschool-age children are most likely to view illness as a punishment for misdeeds. Separation anxiety, although seen in all age-groups, is most common in older infants. Fear of death is typical of older school-age children and adolescents. Adolescents also fear mutilation

A school nurse is preparing a teaching session on safety for parents of school-aged children. What would be an appropriate topic for this age group? a) Providing close supervision to prevent injuries b) Providing drug, alcohol, and sexuality education c) Teaching stress reduction techniques d) Selecting toys for the developmental level

Providing drug, alcohol, and sexuality education Correct Explanation: The school-aged child should be taught drug, alcohol, and sexuality education. Selecting toys for the developmental level applies to infants. Teaching stress reduction techniques applies to adults. Providing close supervision to prevent injuries applies to toddlers.

After assessing a female adolescent, a nurse collects the following data: development of breast tissue, growth spurt in height and weight, appearance of axillary hair, and initiation of menarche. Which stage of sexual development does this data confirm? a) Precocious puberty b) Post pubescence c) Prepubescence d) Pubescence

Pubescence Correct Explanation: Adolescent sexual development is divided into three categories, Prepubescence, pubescence, and post pubescence. Prepubescence would be defined by rounding of the hips, appearance of breast buds, and appearance of downy pubic hair. Pubescence would be defined by development of breast tissue, growth spurt in height and weight, appearance of axillary hair, and initiation of menarche. Post pubescence would be defined by completion of sexual growth and development and fertility. Precocious puberty is puberty that begins prior to age 8 in girls, and age 9 in boys. Certain conditions such as infections, brain abnormalities, tumors, or hormone disorders may cause precocious puberty

The nurse is talking with a group of teens about transmission of human immunodeficiency virus (HIV). What body fluids does the nurse inform them will transmit the virus? Select all that apply. a) Blood b) Vaginal secretions c) Semen d) Urine e) Breast milk

Semen • Breast milk • Blood • Vaginal secretions Correct Explanation: There are only four known body fluids through which HIV is transmitted:blood, semen, vaginal secretions, and breast milk. HIV may be present in saliva, tears, and conjunctival secretions, but transmission of HIV through these fluids has not been implicated, HIV is not found in urine, stool, vomit, or sweat

Parents ask the nurse for advice about handling their 2-year-old's negativism. What is the best recommendation? a) Ignore this behavior because it is a stage the child is going through. b) Punish the child for misbehaving or violating set, strict limits. c) Encourage the grandmother to visit frequently to relieve them. d) Set realistic limits for the child, and then be sure to stick to them.

Set realistic limits for the child, and then be sure to stick to them. Correct Explanation: A characteristic of 2-year-olds is negativism, a response to their developing autonomy. Setting realistic limits is important so that the toddler learns what behavior is and is not acceptable. Ignoring the behavior may lead the child to believe that there are no limits. As a result, the child does not learn appropriate behavior. Having the grandmother visit will give the parents a break, but setting limits is more important to the child's development. Limits need to be realistic to ensure that the child learns appropriate behavior. Limits that are too strict are inappropriate, interfering with learning appropriate behavior.

A client and her partner come to the clinic stating they have been unable to have sexual intercourse. The female client states she has pain and her "vagina is too tight." The client was raped at age 15 years of age. Which nursing problem is most appropriate for this client? a) Risk for Trauma related to fear of vaginal penetration b) Vaginismus related to vaginal constriction c) Dysfunctional Grieving related to loss of self- esteem because of lack of sexual intimacy d) Sexual Dysfunction related to sexual trauma

Sexual Dysfunction related to sexual trauma Correct Explanation: Sexual dysfunction is the problem that is the most appropriate. Dysfunctional grieving because of lack of intimacy is not correct as the couple may have emotional intimacy. The trauma occurred when the female client was 15 years of age thus is not an acute problem. Vaginismus is a medical diagnosis

The nurse is teaching a class about sexuality at a public health clinic. After class the nurse is approached by Cathleen, a 54-year-old female, who eventually discloses to the nurse that she is having a sexual affair with a female partner. The term for an individual's preference for a partner of a particular gender is which of the following? a) Sexual orientation b) Gender role behavior c) Transvestite d) Transsexual

Sexual orientation Correct Explanation: Sexual orientation refers to the preferred gender of the partner of an individual. A transvestite is an individual who desires to take on the role or wear the clothes of the opposite sex. Gender role behavior is the behavior a person conveys about being male or female. A transsexual is a person of a certain biologic gender who has the feelings of the opposite sex.

A client is treated in the clinic for a sexually transmitted infection, and the nurse suspects that the client is at risk for HIV. The physician determines that the client should be tested for the virus. What responsibility does the nurse have? a) The nurse ensures a written consent is obtained prior to testing. b) The nurse will inform the client that the results will have to be reported to the Centers for Disease Control and Prevention (CDC). c) The nurse should send the client to have the blood drawn without informing him about the specific screening test. d) The nurse will call the client with the results of the test.

The nurse ensures a written consent is obtained prior to testing. Correct Explanation: The nurse ensures that a written consent is obtained before testing for human immunodeficiency virus (HIV) and keeps the results of HIV testing confidential. The client should never be tested without his knowledge. The physician will review the results when the client comes in for a follow-up visit. It is not necessary for the nurse to report results to the CDC

The nurse caring for patients in a physician's office takes into consideration developmental stage when assessing sexuality. Which is an example of a developmentally appropriate intervention? a) The nurse teaches parents of a 4-year-old that they may cause anxiety in the child by intolerance of inconsistency of sex-role behavior. b) The nurse states that same-sex preference for relationships in the school-aged child may be related to heterosexual or homosexual tendencies. c) The nurse warns parents of 2-year-old that toilet training should be initiated immediately to prevent compulsive behaviors later on. d) The nurse teaches parents of an 18-month old to discourage self-manipulation of genitals.

The nurse teaches parents of a 4-year-old that they may cause anxiety in the child by intolerance of inconsistency of sex-role behavior. Correct Explanation: The process of human development affects the psychosocial, emotional, and biologic aspects of life, which in turn affect a person's sexuality. The 4 to 6 year-old becomes increasingly aware of their sexual self, demonstrated by enjoyment in exploring body parts of self and playmates, and engaging in masturbation. Parents may cause anxiety in the child by intolerance of inconsistency of sex-role behavior. The nurse should teach the parents of an 18-month old to not discourage self-manipulation of genitals. Parents should allow toddlers to designate their own readiness for toilet training because strict measures may lead to compulsive behaviors later. Same-sex preference for relationships is not related to heterosexual or homosexual tendencies in the school-aged child

The nurse is teaching a class about sexuality at a public health clinic. After class the nurse is approached by Cathleen, a 54-year-old female, who eventually discloses to the nurse that, while she is happily married, she feels like a man inside. Cathleen describes this as feeling "trapped" inside her female body. The term for this behavior is which of the following? a) Transvestite b) Transsexual c) Bisexual d) Gender role behavior

Transsexual Correct Explanation: A transsexual is a person of a certain biologic gender who has the feelings of the opposite sex. A bisexual is a person who finds pleasure with both opposite-sex and same-sex partners. A transvestite is an individual who desires to take on the role or wear the clothes of the opposite sex. Gender role behavior is the behavior a person conveys about being male or female.

Which of the following dinner selections demonstrates an understanding of nutritional therapy used by women to decrease the signs and symptoms of menopause? a) White toast, apple sauce, grilled chicken, and glazed carrots b) Saltine crackers, fruit cocktail, lima beans, and meatloaf c) Corn chips, grapes, lean meat, and baked beans d) Wheat toast, apple slices, broiled chicken breast, and steamed carrots

Wheat toast, apple slices, broiled chicken breast, and steamed carrots Correct Explanation: To decrease the signs and symptoms of menopause, women are encouraged to decrease their fat and caloric intake and increase their intake of whole grains, fiber, fruit, and vegetables. Saltine crackers, white toast, and corn chips are not good sources of fiber. Fruit cocktail, applesauce, and grapes are high in artificial and natural sugars. Meatloaf is high in fat. Glazed carrots and baked beans can be high in sugar content

Which of the following instructions regarding future sexual activity should a nurse give a patient with a vasectomy? a) Use a reliable method of contraception until the physician ensures that sperm no longer are present b) Administer a mild analgesic before any sexual activity c) Sexual activity can resume after three weeks d) Expect some bruising and incisional soreness after every sexual activity for the first 2days

a For a patient with a vasectomy, the nurse advises to use a reliable method of contraception until the physician ensure that sperm no longer are present, which may be determined after 10 or more ejaculations. The patient should resume sexual activity when comfort allows, usually in 1 week. Administer a mild analgesic such as aspirin or acetaminophen for discomfort and not before the sexual activity. The patient should expect some bruising and incisional soreness after the local anesthetic wears off.

Which child should be referred for further assessment regarding language development? a) a 3-year-old who has a vocabulary of 900 words and can make a complete sentence of three or four words b) a 4-year-old is whose speech is understood 50% of the time c) a 1-year-old who has a vocabulary of eight words and can say "mommy" and "daddy" with specific reference to the correct person d) a 2-year-old who has a vocabulary of 300 words and can combine two or three words in a phrase

a 4-year-old is whose speech is understood 50% of the time Correct Explanation: At age 4 a child's speech should be understood most of the time even by people who do not know the child. According to the Denver Developmental Screening Examination, a child age 2 years should have a vocabulary of 300 words, be able to combine two or three words, and ask for what he or she wants by name. By age 3, the child should have a vocabulary of 900 words and can use a complete sentence of three or four words. A 1-year-old has a vocabulary of at least eight words and can reference people and objects.

When assessing speech development, which child should the nurse refer for further examination? a) a 10-month-old who says "dada" and "mama" b) a 4-month-old who laughs out loud c) an 18-month-old who only says "no" d) a 1-year-old who says 3 to 5 words

an 18-month-old who only says "no" Correct Explanation: An 18-month-old child should be able to say 10 or more words. Lack of speech development may indicate a lack of social stimulation, a hearing deficiency, or developmental delay. Referring the child for an evaluation may increase the child's chance of reaching the child's potential. A 4-month-old child with a healthy central nervous system and normal mental development should be able to laugh out loud if the child's environment has been caring and the child's needs are met safely and consistently. Children at age 10 months should be able to say the words "dada" and "mama" in response to the appropriate person. A 1-year-old child should have the ability to speak three to five words plus "mama" and "dada."

A 4-year-old child continues to come to the nurses' station after being told children are not allowed there. What behavior is the child exhibiting? a) resistive behavior b) exaggerated stress behavior c) attention-seeking behavior d) aggressive behavior

attention-seeking behavior Correct Explanation: The child wants attention from the nurse, even if the behavior is met by a negative response. Aggression, resistance against authority, and exaggerated stress are behaviors that can be associated with a 4-year-old. However, coming to the nurses' station after being told not to do so is not an example of these behaviors

A client is in the last trimester of pregnancy. The nurse should instruct her to notify her primary health care provider immediately if she notices: a) hemorrhoids. b) increased vaginal mucus. c) blurred vision. d) dyspnea on exertion.

blurred vision. Correct Explanation: Blurred vision or other visual disturbances, excessive weight gain, edema, and increased blood pressure may signal severe preeclampsia. This condition may lead to eclampsia, which has potentially serious consequences for the client and fetus. Although hemorrhoids may be a problem during pregnancy, they don't require immediate attention. Increased vaginal mucus and dyspnea on exertion are expected as pregnancy progresses.

A client wants to avoid methods of birth control that contain estrogen. Which method would be the nurse recommend? a) birth control patch b) combined hormonal oral contraceptive c) depot medroxyprogesterone acetate injection d) etonogestrel/ethinyl estradiol vaginal ring

depot medroxyprogesterone acetate injection Correct Explanation: Birth control methods that contain estrogen increase risk for clotting disorders especially in women over the age of 35 years who smoke or who have had a a previous clotting problem. Depot medroxyprogesterone acetate (DMPA) injections contain progesterone, but no estrogen. Combined hormonal contraceptives, vaginal rings, and the birth control patch all contain estrogen.

A 2-year-old always puts his teddy bear at the head of his bed before he goes to sleep. The parents ask the nurse if this behavior is normal. The nurse should explain to the parents that toddlers use ritualistic patterns to: a) establish a sense of identity. b) establish control over adults in their environment. c) establish sequenced patterns of learning behavior. d) establish a sense of security.

establish a sense of security. Correct Explanation: Toddlers establish ritualistic patterns to feel secure, despite inconsistencies in their environment. Establishing a sense of identity is the developmental task of the adolescent. The toddler's developmental task is to use rituals and routines to help in making autonomy easier to accomplish. Ritualistic patterns do involve patterns of behavior, but they are not utilized to develop learning behaviors

When the nurse is teaching a group of parents about common childhood problems, a parent asks, "Why are children more likely to develop ear infections than adults are?" The nurse bases the response to this question on the understanding that the key anatomic difference between adults and children is due to which structure? a) eustachian tubes b) ear canals c) nasopharynx d) tympanic membranes

eustachian tubes Correct Explanation: In infants and young children, the eustachian tubes are short and lie in a relatively horizontal position. This anatomic position favors the development of otitis media because it is easy for materials from the nasopharynx to enter the tubes. Although bacteria may be present in the nasopharynx, this does not affect middle ear function. The size of the ear canal has no impact on the increased number of ear infections in children. An intact tympanic membrane prevents bacteria from entering the middle ear from the external ear canal. The tympanic membrane changes appearance with an ear infection, but its structure does not predispose infants and young children to ear infection.

On a home visit following discharge from the hospital after treatment for severe gastroenteritis, the parent tells the nurse that a toddler answers "No!" and is difficult to manage. After discussing this further with the parent, the nurse explains that the child's behavior is most likely the result of which factor? a) usual lack of interest in everything b) inherited personality trait c) expression of individuality d) beginning leadership skills

expression of individuality Correct Explanation: The "no" behavior demonstrated by a toddler is typical of this age group as the child attempts to be self-assertive as an individual. The negativism does not demonstrate an inherited personality trait or disinterest. Rather, it reflects the developmental task of establishing autonomy. The toddler is attempting to exert control over the environment. It is too early to assess leadership qualities in a toddler

A 10-year-old boy tells his mother that he is going to be just like his father when he grows up. According to Freud, what stage of development is this child experiencing? a) phallic b) genital c) latency d) anal

latency Explanation: The latency stage marks the transition to the genital stage during adolescence. Increasing sex-role identification with the parent of the same sex prepares the child for adult roles and relationships.

A 22-year-old client tells the nurse that she and her husband are trying to conceive a baby. When teaching the client about reducing the incidence of neural tube defects, the nurse would emphasize the need for increasing the intake of which foods? Select all that apply. a) strawberries b) sunflower seeds c) leafy green vegetables d) lentils e) beans f) milk

leafy green vegetables • strawberries • beans • sunflower seeds • lentils Explanation: The pregnancy requirement for folic acid is 400 to 800 micrograms/day. Major sources of folic acid include leafy green vegetables, strawberries and oranges, beans, particularly black and kidney beans, sunflower seeds, and lentils. Milk and fats contain no folic acid

To assess the development of a 1-month-old, the nurse asks the parent if the infant is able to: a) roll from back to side. b) hold a rattle briefly. c) lift head from prone position. d) smile and laugh out loud.

lift head from prone position. Correct Explanation: A 1-month-old infant is usually able to lift the head from a prone position. The full-term infant with no complications has probably been able to do this since birth. Smiling and laughing is expected behavior at 2 to 3 months. Rolling from back to side and holding a rattle are characteristics of a 4-month-old.

The parent tells the nurse that an 8-year-old child is continually telling jokes and riddles to the point of driving the other family members crazy. The nurse should explain this behavior is a sign of: a) inappropriate peer influence. b) excessive television watching c) mastery of language ambiguities. d) inadequate parental attention.

mastery of language ambiguities. Explanation: School-age children delight in riddles and jokes. Mastery of the ambiguities of language and of sentence structure allows the school-age child to manipulate words, and telling riddles and jokes is a way of practicing this skill. Children who suffer from inadequate attention from parents tend to demonstrate abnormal behavior. Peer influence is less important to school-age children, and while the child may learn the joke from a friend, he is telling the joke to master language. Watching television does not influence the extent of joke telling

A mother tells the nurse that her 4 1/2-year-old child "does not seem to know the difference between right and wrong." This behavior is typical of which levels as described by Kohlberg's theory of levels of moral development? a) conventional b) autonomous c) preconventional d) principled

preconventional Correct Explanation: The preconventional level of Kohlberg's stages of moral development is typical of the preschool-aged child. Stage 1 behaviors of this preconventional level have a punishment-obedience orientation. Children at this stage avoid punishment and avoid those who have power. Autonomous, or postconventional, is the third stage of moral development as described by Kohlberg. These children are concerned with defining values and principles. The conventional level of morality development pertains to children aged 7 to 12 years who are concerned with loyalty and conformity. Principled is another name for the autonomous or postconventional stage, the third stage of moral development as described by Kohlberg. These children are concerned with defining values and principles.

A toddler who has been treated for a foreign body aspiration begins to fuss and cry when the parents attempt to leave the hospital for an hour. The parents will be returning to take the toddler home. As the nurse tries to take the child out of the crib, the child pushes the nurse away. The nurse interprets this behavior as indicating which stage of separation anxiety? a) detachment b) regression c) despair d) protest

protest Correct Explanation: Young children have specific reactions to separation and hospitalization. In the protest stage, the toddler physically and verbally attacks anyone who attempts to provide care. Here, the child is fussing and crying and visibly pushes the nurse away. In the despair stage, the toddler becomes withdrawn and obviously depressed (e.g., not engaging in play activities and sleeping more than usual). Regression is a return to a developmentally earlier phase because of stress or crisis (e.g., a toddler who could feed himself before this event is not doing so now). Denial or detachment occurs if the toddler's stay in the hospital without the parent is prolonged because the toddler settles in to the hospital life and denies the parents' existence (e.g., not reacting when the parents come to visit)

A 2 1/2-year-old child and his 2-month-old sibling are brought to the clinic by their father, who explains that the older child says "no" whenever asked to do something. The nurse should explain that the negativism demonstrated by toddlers is frequently an expression of which characteristic? a) pursuit of autonomy b) need to expend excess energy c) separation anxiety d) sibling rivalry

pursuit of autonomy Correct Explanation: According to Erikson, the developmental task of toddlerhood is acquiring a sense of autonomy while overcoming a sense of doubt and shame. Characteristics of negativism and ritualism are typical behaviors in this quest for autonomy. The toddler commonly does the opposite of what others request. Hyperactivity, or the need to expend excess energy, is a typical behavior that may be demonstrated by a toddler; separation anxiety and siblings rivalry may also be demonstrated by the toddler. However, none of these three behaviors is the basis for the toddler's negativism.

The mother asks the nurse for advice about discipline for her 18-month-old. Which discipline strategy should the nurse suggest that the mother use? a) reasoning b) time-out c) reprimand d) spanking

time-out Explanation: Time-out is the most appropriate discipline for toddlers. It helps to remove them from the situation and allows them to regain control. Structuring interactions with 3-year-olds helps minimize unacceptable behavior. This approach involves setting clear and reasonable rules and calling attention to unacceptable behavior as soon as it occurs. Reprimanding a young child can reinforce undesirable behavior over time because it provides attention. Physical punishment, such as spanking, has limited effectiveness and serious negative effects. Reasoning is more appropriate for older children, such as preschoolers and those older, especially when moral issues are involved. Unfortunately, reasoning combined with scolding often takes the form of shame or criticism and children take such remarks seriously, believing that they are "bad.


संबंधित स्टडी सेट्स

Cognitive Psychology Book Questions Chapter 9

View Set

chapter 9 (education and training)

View Set

Principals of Management Chapter 13

View Set